2017 (Paper 2) Flashcards

1
Q

Lady with falciparum. Other than doxycycline, what other abx would you give her?

Options:

Chloroquine

Quinine

Proguanil

Pyrimethamine

A

Quinine

Non-artemisinin based combinations: Quinine plus tetracycline/doxycycline

Feature of severe malaria

  • schizonts on a blood film
  • parasitaemia > 2%
  • hypoglycaemia
  • acidosis
  • temperature > 39 °C
  • severe anaemia
  • complications:
    • cerebral malaria: seizures, coma
    • acute renal failure: blackwater fever, secondary to intravascular haemolysis, mechanism unknown
    • acute respiratory distress syndrome (ARDS)
    • hypoglycaemia
    • disseminated intravascular coagulation (DIC)

Uncomplicated falciparum malaria

  • strains resistant to chloroquine are prevalent in certain areas of Asia and Africa
  • the 2010 WHO guidelines recommend artemisinin-based combination therapies (ACTs) as first-line therapy
  • examples include artemether plus lumefantrine, artesunate plus amodiaquine, artesunate plus mefloquine, artesunate plus sulfadoxine-pyrimethamine, dihydroartemisinin plus piperaquine

Severe falciparum malaria

  • a parasite counts of more than 2% will usually need parenteral treatment irrespective of clinical state
  • intravenous artesunate is now recommended by WHO in preference to intravenous quinine
  • if parasite count > 10% then exchange transfusion should be considered
  • shock may indicate coexistent bacterial septicaemia - malaria rarely causes haemodynamic collapse
How well did you know this?
1
Not at all
2
3
4
5
Perfectly
2
Q

Which of the DMARDs cause retinopathy?

Options:

Hydroxychloroquine

Prednisolone

Sulfasalazine

Leflunomide

A

Hydroxychloroquine

Rheumatoid arthritis drug side effects

  • Hydroxychloroquine
    • Retinopathy
    • Corneal deposits
  • Methotrexate
    • Myelosuppression
    • Liver cirrhosis
    • Pneumonitis
  • Sulfasalazine
    • Rashes
    • Oligospermia
    • Heinz body anaemia
    • Interstitial lung disease
  • Leflunomide
    • Liver impairment
    • Interstitial lung disease
    • Hypertension
  • Prednisolone
    • Cushingoid features
    • Osteoporosis
    • Impaired glucose tolerance
    • Hypertension
    • Cataracts
  • Gold
    • Proteinuria
  • Penicillamine
    • Proteinuria
    • Exacerbation of myasthenia gravis
  • Etanercept
    • Demyelination
    • Reactivation of tuberculosis
  • Infliximab
    • Reactivation of tuberculosis
  • Adalimumab
    • Reactivation of tuberculosis
  • Rituximab
    • Infusion reactions are common
  • NSAIDs (e.g. naproxen, ibuprofen)
    • Bronchospasm in asthmatics
    • Dyspepsia/peptic ulceration
How well did you know this?
1
Not at all
2
3
4
5
Perfectly
3
Q

Which of the DMARDs cause azoospermia and bone marrow suppression?

Options:

Sulfasalazine

Azathioprine

Methotrexate

Leflunomide

A

Sulfasalazine

  • Hydroxychloroquine
    • Retinopathy
    • Corneal deposits
  • Methotrexate
    • Myelosuppression
    • Liver cirrhosis
    • Pneumonitis
  • Sulfasalazine
    • Rashes
    • Oligospermia
    • Heinz body anaemia
    • Interstitial lung disease
  • Leflunomide
    • Liver impairment
    • Interstitial lung disease
    • Hypertension
  • Prednisolone
    • Cushingoid features
    • Osteoporosis
    • Impaired glucose tolerance
    • Hypertension
    • Cataracts
  • Gold
    • Proteinuria
  • Penicillamine
    • Proteinuria
    • Exacerbation of myasthenia gravis
  • Etanercept
    • Demyelination
    • Reactivation of tuberculosis
  • Infliximab
    • Reactivation of tuberculosis
  • Adalimumab
    • Reactivation of tuberculosis
  • Rituximab
    • Infusion reactions are common
  • NSAIDs (e.g. naproxen, ibuprofen)
    • Bronchospasm in asthmatics
    • Dyspepsia/peptic ulceration
How well did you know this?
1
Not at all
2
3
4
5
Perfectly
4
Q

Lady with tremor, weight loss, proptosis and exophthalmos. Has ophthalmoplegia. What is likely cause?

Options:

Rectus muscle thickening

Retrobulbar tumour

Cavernous sinus syndrome

A

Rectus muscle thickening

as seen in thyroid eye disease

Pathophysiology: it is thought to be caused by an autoimmune response against an autoantigen, possibly the TSH receptor → retro-orbital inflammation –> glycosaminoglycan and collagen deposition in the muscles

(frequently: inferior rectus > medial rectus > superior rectus) sparing their tendinous insertions, and is usually bilateral and symmetrical.

Features

  • the patient may be eu-, hypo- or hyperthyroid at the time of presentation
  • exophthalmos
  • conjunctival oedema
  • optic disc swelling
  • ophthalmoplegia
  • inability to close the eye lids may lead to sore, dry eyes. If severe and untreated patients can be at risk of exposure keratopathy

Prevention

  • smoking is the most important modifiable risk factor for the development of thyroid eye disease
  • radioiodine treatment may increase the inflammatory symptoms seen in thyroid eye disease. In a recent study of patients with Graves’ disease around 15% developed, or had worsening of, eye disease. Prednisolone may help reduce the risk

Management

  • topical lubricants may be needed to help prevent corneal inflammation caused by exposure
  • steroids
  • radiotherapy
  • surgery
How well did you know this?
1
Not at all
2
3
4
5
Perfectly
5
Q

69 year old guy with two previous TIAs and AF. What do you start him on?

Option:

Warfarin

Aspirin

Clopidogrel

Aspirin + Modified release dipyridamol

A

Warfarin or NOAC (Dabigatran, Rivaroxaban, Apixaban)

nb Direct oral anticoagulants (direct thrombin/Xa inhibitors) are used in the prevention of stroke secondary to non-valvular AF

Post ischaemic stroke prophylaxis:

  • clopidogrel is now recommended by NICE ahead of combination use of aspirin plus modified release (MR) dipyridamole in people who have had an ischaemic stroke
  • aspirin plus MR dipyridamole is now recommended after an ischaemic stroke only if clopidogrel is contraindicated or not tolerated, but treatment is no longer limited to 2 years’ duration
  • MR dipyridamole alone is recommended after an ischaemic stroke only if aspirin or clopidogrel are contraindicated or not tolerated, again with no limit on duration of treatment
  • Stoke secondary to AF:
    • following a stroke or TIA warfarin or a DOAC should be given as the anticoagulant of choice.
      • If valvular AF, Warfarin must be used
      • Aspirin/dipyridamole should only be given if needed for the treatment of other comorbidities
    • in acute stroke patients, in the absence of haemorrhage, anticoagulation therapy should be commenced after 2 weeks. If imaging shows a very large cerebral infarction then the initiation of anticoagulation should be delayed
How well did you know this?
1
Not at all
2
3
4
5
Perfectly
6
Q

Lady with hx of AF presents to A&E with fast AF, pulmonary oedema and peripheral oedema. What do you give initially?

Options:

Furosemide and warfarin

Furosemide and Ramipril

A

DC cardioversion (if an option)

Cardioversion indicated in Peri-arrest rhythms: tachycardia

  • shock: hypotension (systolic blood pressure < 90 mmHg), pallor, sweating, cold, clammy extremities, confusion or impaired consciousness
  • syncope
  • myocardial ischaemia
  • heart failure

Furosemide and ?warfarin

  • nb.acute pulmonary edema may increase sensitivity to ACE inhibition or angiotensin blockade, including risks of hypotension and renal dysfunction*
  • warfarin will initiall decrease the INR (increase risk of clotti**ng) – it is often recommended that heparin should be administered in the ED if a AF patient is going to be admitted to hospital for expedited elective cardioversion (AF duration < 48 hours, or AF duration > 48 hours with a negative TEE study)*

Management options in acute heart failure include:

  • oxygen
    • diuretics
    • opiates
    • vasodilators
    • inotropic agents
    • CPAP
    • ultrafiltration
    • mechanical circulatory assistance: e.g. intra-aortic balloon counterpulsation or ventricular assist devices

Drug management of heart failure:

  1. first-line= ACE-inhibitor + beta-blocker
  2. second-line= aldosterone antagonist, angiotensin II receptor blocker or a hydralazine in combination with a nitrate
  3. third line= cardiac resynchronisation therapy or digoxin* or ivabradine (in pt on ACE-inhibitor, beta-blocker + aldosterone antagonist, with HR > 75/min and a left ventricular fraction < 35%)
  • diuretics should be given for fluid overload
  • offer annual influenza vaccine
  • offer one-off** pneumococcal vaccine
How well did you know this?
1
Not at all
2
3
4
5
Perfectly
7
Q

30 something man with joint pain - sacroilitis and distal interphalangeal joint pain?

Options:

Ank spond

Psoriatic arthritis

Rheumatoid arthritis.

A

Psoriatic arthropathy

correlates poorly with cutaneous psoriasis and often precedes the development of skin lesions. Around 10-20% percent of patients with skin lesions develop an arthropathy with males and females being equally affected

Types*

  • rheumatoid-like polyarthritis: (30-40%, most common type)
  • asymmetrical oligoarthritis: typically affects hands and feet (20-30%)
  • sacroilitis
  • DIP joint disease (10%)
  • arthritis mutilans (severe deformity fingers/hand, ‘telescoping fingers’)

Management

  • treat as rheumatoid arthritis
  • but better prognosis

nb ankylosing spondylitis: affects axial spine

How well did you know this?
1
Not at all
2
3
4
5
Perfectly
8
Q

Guy on prednisolone presents with darkened red reflex and problems with night vision. What does he have?

A

Cataracts

opacities in lens, cornea or vitreous results in loss of red reflex

Presentation

  • Increasing myopia
  • Blurred vision → gradual visual loss
  • Dazzling in sunshine / bright lights
  • Monocular diplopia

Cause

Majority

  • age related
  • UV light

Systemic

  • diabetes mellitus
  • steroids
  • infection (congenital rubella)
  • metabolic (hypocalcaemia, galactosaemia)
  • myotonic dystrophy
  • Down’s syndrome

Ocular

  • trauma
  • uveitis
  • high myopia
  • topical steroids

Classification

  • Nuclear: change lens refractive index, common in old age
  • Polar: localized, commonly inherited, lie in the visual axis
  • Subcapsular: due to steroid use, just deep to the lens capsule, in the visual axis
  • Dot opacities: common in normal lenses, also seen in diabetes and myotonic dystrophy
How well did you know this?
1
Not at all
2
3
4
5
Perfectly
9
Q

Lady from some Pakistan or somewhere with HTN controlled on amlodipine and BP of 150/90 has retinal findings: blot haemorrhages and yellow deposits (hard exudates I think) on macula. What is this?

Options:

Hypertensive retinopathy

Diabetic retinopathy

A

NOTE: Drusen seen in age related macular degeneration

Background diabetic retinopathy

  • microaneurysms (dots)
  • blot haemorrhages (<=3)
  • hard exudates

Pre-proliferative retinopathy

  • cotton wool spots (soft exudates; ischaemic nerve fibres)
  • > 3 blot haemorrhages
  • venous beading/looping
  • deep/dark cluster haemorrhages
  • more common in Type I DM, treat with laser photocoagulation

Proliferative retinopathy

  • retinal neovascularisation - may lead to vitrous haemorrhage
  • fibrous tissue forming anterior to retinal disc
  • more common in Type I DM, 50% blind in 5 years

Maculopathy

  • based on location rather than severity, anything is potentially serious
  • hard exudates and other ‘background’ changes on macula
  • check visual acuity
  • more common in Type II DM
How well did you know this?
1
Not at all
2
3
4
5
Perfectly
10
Q

Footballer inverts his ankle and presents with foot pain, surprised he has no ankle pain. What does he have?

Options:

Fractured base of 5th metatarsal

Damaged ligaments

A

Fractured base of 5th metatarsal

is caused by forcible inversion of the foot in plantar flexion

Associated with Lisfranc injury

How well did you know this?
1
Not at all
2
3
4
5
Perfectly
11
Q

Guy jumps and lands on his knee (repeated question)

Options:

medial meniscus tear

patella fracture

damage to ACL

A

patella fracture

How well did you know this?
1
Not at all
2
3
4
5
Perfectly
12
Q

Pulmonary adenocarcinoma Ca met to liver via what route?

Options:

Transcoelomic

Direct invasion

Haem

Lymph

A

Heamatogenous

Spread:

  • local: to pleura, left recurrent laryngeal and phrenic nerve, pericardium, oesophagus, sympathetic chain, and brachial plexus
  • Lymphatic: to mediastinal and cervical nodes. Compression of the SVC by massive mediastinal node involvement produces gross oedema and cyanosis of the face and upper limbs
  • Blood: to bone, brain, liver and adrenals
  • Transcoelomic: pleural seeding and effusion

Cancers which matastasise heamatogenously = Folicular, Renal Cell Ca, Hepatocellular carcinoma

  • *Hematogenous–Hepatoma**
  • *S**preading – Sarcoma
  • *Cancers –Choriocarcinoma**
  • *R**eign – Renal cell carcinoma
  • *F**oolishly – Follicular carc. of the thyroid
How well did you know this?
1
Not at all
2
3
4
5
Perfectly
13
Q

Lady playing squash suddenly runs, hears a crack from behind ankle then pain whenever she tries to plantar flex. What happened?

options:

Ruptured Achilles

Fractured talus

Fractured calcaneus

A

Achilles tendon rupture

..suspected if the person describes the following whilst playing a sport or running; an audible ‘pop’ in the ankle, sudden onset significant pain in the calf or ankle or the inability to walk or continue the sport.

An examination should be conducted using Simmond’s triad, to help exclude Achilles tendon rupture. This can be performed by asking the patient to lie prone with their feet over the edge of the bed. The examiner should look for an abnormal angle of declination; Achilles tendon rupture may lead to greater dorsiflexion of the injured foot compared to the uninjured limb. They should also feel for a gap in the tendon and gently squeeze the calf muscles if there is an acute rupture of the Achilles tendon the injured foot will stay in the neutral position when the calf is squeezed.

An acute referral should be made to an orthopaedic specialist following a suspected rupture.

How well did you know this?
1
Not at all
2
3
4
5
Perfectly
14
Q

Middle aged lady presenting with atypical chest pain – what would be used to decide pre test probability she has ischaemic heart disease?

Options:

Epidemiological evidence of something

Case control study on atypical chest pain RFs

Clinical experience

Exercise ECG

A

Epidemiological evidence of something

Evidence based medicine works by deciding pre-test probability of something and then doing tests and changing the probability of that thing accordingly.

Pretest Probability

  • Definition
    • Pretest Probability is defined as the probability of a patient having the target disorder before a diagnostic test result is known. It represents the probability that a specific patient, say a middle-aged man, with a specific past history, say hypertension and cigarette smoking, who presents to a specific clinical setting, like Accident and Emergency, with a specific symptom complex, say retrosternal chest pressure, dyspnoea and diaphoresis, has a specific diagnosis, such as acute myocardial infarction.
  • Application
    • The pretest probability is especially useful for four things:
  1. interpreting the results of a diagnostic test,
  2. selecting one or more diagnostic tests
  3. choosing whether to start therapy:
    • * a) without further testing (treatment threshold);
    • * b) while awaiting further testing;
  4. deciding whether it’s worth testing at all (test threshold)
  • Calculation
    • The probability of the target disorder, usually abbreviated P(D+), can be calculated as the proportion of patients with the target disorder, out of all the patients with the symptoms(s), both those with and without the disorder:
      • P(D+) = D+ / (D+ + D-)
    • where D+ indicates the number of patients with target disorder, D- indicates the number of patients without target disorder, and P(D+) is the probability of the target disorder.
  • Example:
    • A group of investigators in North America studied the underlying diseases found in patients presenting to a primary care setting with persistent dizziness. They studied a total of 100 dizzy patients, finding 16 had dizziness from psychiatric conditions.
    • Thus, 16 / 100, or 16 per cent, is the estimate of of disease probability for psychiatric conditions from this study.
    • So, if this information proves valid and applicable to your practice, if IF one of your patients presents to a primary care setting with persistent dizziness, the P(D+) of a psychiatric cause might be estimated at 16 per cent.
How well did you know this?
1
Not at all
2
3
4
5
Perfectly
15
Q

Person with 4cm head of pancreas ca that has invaded mesenteric vessels. How do you manage?

options

ERCP and biliary stent

PTC drainage

Whipple’s

Vit k

Abx

A

ERCP and biliary stent

Management

  • less than 20% are suitable for surgery at diagnosis
  • Resectable:
    • a Whipple’s resection (pancreaticoduodenectomy) is performed for resectable lesions in the head of pancreas.
      • Side-effects of a Whipple’s include dumping syndrome and peptic ulcer disease
    • adjuvant chemotherapy is usually given following surgery
  • Palliation:
    • ERCP with stenting is often used for palliation
How well did you know this?
1
Not at all
2
3
4
5
Perfectly
16
Q

Son of 75 year old lady requests home visit for mother who has recent behavioural changes. Sometimes gets confused and sees people in the room who aren’t there. Recent loss of appetite. Cause?

options:

Lewy body dementia

Alzheimer’s

Acute Confusional State

Depression with psychosis

A

Acute confusional state

is also known as delirium or acute organic brain syndrome. It affects up to 30% of elderly patients admitted to hospital.

Features - wide variety of presentations

  • memory disturbances (loss of short term > long term)
  • may be very agitated or withdrawn
  • disorientation
  • mood change
  • visual hallucinations
  • disturbed sleep cycle
  • poor attention

(if not an option, potentially LBD)

How well did you know this?
1
Not at all
2
3
4
5
Perfectly
17
Q

Guy post-op with delirium and morphine epidural in situ, what is best initial management?

Options:

Put in well lit side room

Haloperidol

Discontinue morphine epidural

A

Put in a well lit side room

Management of Acute Confusional State

  • treatment of underlying cause
  • modification of environment
  • haloperidol 0.5 mg as the first-line sedative or olanzapine
How well did you know this?
1
Not at all
2
3
4
5
Perfectly
18
Q

Guy with short history of back pain and painless black lesions on his feet appear. What ix do you do?

Options:

Arteriogram

CT abdo

A

CT abdo with contrast

Abdominal aortic aneurysms occur primarily as a result of the failure of elastic proteins within the extracellular matrix. Aneurysms typically represent dilation of all layers of the arterial wall. Most aneurysms are caused by degenerative disease. After the age of 50 years the normal diameter of the infrarenal aorta is 1.5cm in females and 1.7cm in males. Diameters of 3cm and greater, are considered aneurysmal. The pathophysiology involved in the development of aneurysms is complex and the primary event is loss of the intima with loss of elastic fibres from the media. This process is associated with, and potentiated by, increased proteolytic activity and lymphocytic infiltration.

symptomatic

  • pain in the abdomen, loin or lower back
    • abdominal pain may spread to the back
    • require urgent referral for investigation and management because they may indicate impending rupture
  • patient may feel a pulsatile mass in the abdomen
  • features of limb ischemia - caused by distal embolisation
  • lower extremity oedema
    • rarely seen caused by compression of the inferior vena cava

Investigations

  • ultrasonography
    • non invasive, low cost method with the sensitivity and specificity close to 100%
    • measures the size of the aorta in longitudinal as well as in anteroposterior and transverse directions
    • modality of choice for initial assessment, follow up, screening and surveillance
    • should have a low threshold for arranging abdominal ultrasonography in patients at risk.
  • CT
    • can visualise
      • the proximal neck (the transition between the normal and aneurysmal aorta)
      • any extensions of the aneurysm (to the iliac areries)
      • patency of visceral arteries
      • the presence of blood within the thrombus (crescent sign) - is considered a predictive marker of imminent rupture
    • helical CT and CT angiography can provide additional anatomical details - useful if an endovascular procedure is considered
  • resonance angiography (MRA)
    • safer than conventional arteriography since it does not use nephrotoxic contrast material
  • plain radiography
    • calcified aneurysmal wall may be seen in some cases
How well did you know this?
1
Not at all
2
3
4
5
Perfectly
19
Q

Guy presenting to GP with nocturia. PSA 18, urinalysis trace blood and protein, urea and cr mildly elevated. What do you do?

Options:

Refer routinely to uro

Refer urgently to uro

Refer routinely to renal

Refer urgently to renal

A

Refer urgently to uro

refer men using a suspected cancer pathway referral (for an appointment within 2 weeks) for prostate cancer if their PSA levels are above the age-specific reference range

Management Programme are as follows:

  • aged 50-59 >= 3.0 ng/ml;
  • aged 60-69 >= 4.0 ng/ml;
  • aged 70 and over > 5.0 ng/ml.
  • (Note that there are no age-specific reference ranges for men over 80 years. Nearly all men of this age have at least a focus of cancer in the prostate. Prostate cancer only needs to be diagnosed in this age group if it is likely to need palliative treatment.)

exclude urinary infection before PSA testing. Postpone the PSA test for at least 1 month after treatment of a proven urinary infection (indicated by nitrite and leucocyte on urine dip)

How well did you know this?
1
Not at all
2
3
4
5
Perfectly
20
Q

?75 year old lady with 2.5cm firm breast lump not tethered to skin and no skin changes. Daughter 40 year old just had benign breast cyst diagnosed. What does the old lady have?

Options:

Ductal carcinoma

Breast cyst

Lobular carcinoma in situ

A

Ductal carcinoma

  • Age, solitary, firm lump – increases likelihood of cancer vs benign cyst. Ductal (70%) more common than lobullary, proliferative breast disease with atypia = risk factor

Breast cyst

  • 7% of all Western females will present with a breast cyst
  • Usually presents as a smooth discrete lump (may be fluctuant)
  • Small increased risk of breast cancer (especially if younger)

Commonest Single Breast Lumps

  • Fibroadenoma
  • Cyst
  • Fat necrosis
  • Cancer

Features of a Malignant Lump

  • Irregular, nodular surface
  • Poorly defined edge
  • Hard / scirrhous consistency
  • Painless
  • Fixation to skin or chest wall
  • Nipple involvement

In this patient AGE makes Ca more likely?

How well did you know this?
1
Not at all
2
3
4
5
Perfectly
21
Q

45 year old lady with spontaneous dark brown nipple discharge. Examination reveals only one duct producing discharge. What does she have?

options:

Intraductal papilloma

Duct ectasia

A

Intraductal papilloma

  • Commoner in younger patients
  • May cause blood stained discharge originating from a single duct
  • There is usually no palpable lump
  • NO risk of malignancy
  • Growth of papilloma in a single duct

Duct ectasia

  • Dilatation breast ducts.
  • Most common in menopausal women
  • Discharge typically thick and green in colour, which may be from single or multiple ducts (usually present age >50 years)
  • Most common in smokers
How well did you know this?
1
Not at all
2
3
4
5
Perfectly
22
Q

Tall 28 year old man with radiofemoral delay and hypertension. BP 210/110. Has rib notching. What is underlying diagnosis?

Options:

Coarctation

Marfan’s

ED

A

Coarctation of the aorta

…describes a congenital narrowing of the descending aorta

more common in males (despite association with Turner’s syndrome)
Features:

  • infancy: heart failure
  • adult: hypertension
  • radio-femoral delay
  • mid systolic murmur, maximal over back
  • apical click from the aortic valve
  • notching of the inferior border of the ribs (due to collateral vessels) is not seen in young children

Associations

  • Turner’s syndrome
  • bicuspid aortic valve
  • berry aneurysms
  • neurofibromatosis

Marfans is associated with the following cardiac abnormalities:

  • Aortic aneurysm and dissection
  • Aortic root dilatation → regurgitation
  • MV prolapse ± regurgitation
How well did you know this?
1
Not at all
2
3
4
5
Perfectly
23
Q

The post MI papillary muscle rupture question (repeated again)

Revision Qs:

How will this present?

List the complications of an MI

A

Acute mitral regurgitation

More common with infero-posterior infarction and may be due to ischaemia or rupture of the papillary muscle. An early-to-mid systolic murmur is typically heard and late systolic click. Patients are treated with vasodilator therapy but often require emergency surgical repair.

Death Passing PRAED st.

  • Death: VF, LVF, CVA
  • Pump Failure
  • Pericarditis
  • Rupture: myomalacia cordis
    • Cardiac tamponade
    • Papillary muscle / chordae → MR
    • Septum
  • Arrhythmias
    • Tachycardias
      • SVT
      • Ventricular T/F
    • Bradycardia
      • sinus brady
      • AV block
      • Ventricular brady
  • Aneurysm: ventricular
  • Dressler’s Syndrome: pleuro-pericarditis
How well did you know this?
1
Not at all
2
3
4
5
Perfectly
24
Q

Young guy vomiting after a night out presents with chest and epigastric pain, left sided pleural effusion, subcutaneous emphysema. What happened?

A

Oesophageal rupture

Boerhaave syndrome: Severe vomiting → oesophageal rupture

Defined as: Complete disruption of the oesophageal wall in absence of per-existing pathology. Left postero-lateral oesophageal is commonest site (2-3cm from OG junction). Suspect in patients with severe chest pain without cardiac diagnosis and signs suggestive of pneumonia without convincing history, where there is history of vomiting. Erect CXR shows infiltrate or effusion in 90% of cases

  • Other:*
  • Mallory-Weiss syndrome: Severe vomiting → painful mucousal lacerations at the gastroesophageal junction resulting in haematemesis. Common in alcoholics*
  • Plummer-Vinson syndrome: Triad of: dysphagia (secondary to oesophageal webs) + glossitis + iron-deficiency anaemia. Treatment includes iron supplementation and dilation of the webs*
How well did you know this?
1
Not at all
2
3
4
5
Perfectly
25
Q

Sickle cell girl with severe back pain (?) What do you do first?

Options:

Exchange transfusion

Epidural anaesthesia

Paracetamol/ibuprofen

Fluid bolus

A

Fluid bolus

Sickle cell anaemia is characterised by periods of good health with intervening crises
Four main types of crises are recognised:

  1. Thrombotic crises/Vaso-occlusive ‘painful’ crisis:
    • precipitated by infection, dehydration, deoxygenation
    • infarcts occur in various organs including the bones (e.g. avascular necrosis of hip, hand-foot syndrome in children, lungs, spleen and brain
    • may cause mesenteric ischaemia, mimicking an acute abdomen
  2. Sequestration crises
    • sickling within organs such as the spleen or lungs causes pooling of blood with worsening of the anaemia
    • acute chest syndrome: dyspnoea, chest pain, pulmonary infiltrates, low pO2 - the most common cause of death after childhood
  3. Aplastic crises
    • caused by infection with parvovirus
    • sudden fall in haemoglobin
  4. Haemolytic crises
    • rare
    • fall in haemoglobin due an increased rate of haemolysis

Give prompt, generous analgesia, eg IV opiates (see p576).
• Crossmatch blood. FBC, reticulocytes, blood cultures, MSU ± CXR if T° or chest signs.
• Rehydrate with IVI and keep warm. Give O2 by mask if Pa O2 or O2 sats <95%.
• ‘Blind’ antibiotics (eg cephalosporin, p382) if T° >38°, unwell, or chest signs.
• Measure PCV, reticulocytes, liver, and spleen size twice daily.
• Give blood transfusion if Hb or reticulocytes fall sharply. Match blood for the blood group antigens Rh(C, D, E) and Kell, to prevent alloantibody formation. This helps oxygenation, and is as good as exchange transfusion (reserved for those who are rapidly worsening: it is a process where blood is removed and donor blood is given in stages). 40 Indications: severe chest crisis, suspected CNS event or multiorgan failure—when the proportion of HbS should be reduced to <30%.

Mx Acute Crises

  • General
    • Analgesia: opioids IV
    • Good hydration
    • O2
    • Keep warm
  • Ix
    • FBC, U+E, reticulocytes, cultures
    • Urine dip
    • CXR
  • Rx
    • Blind Abx: e.g. ceftriaxone
    • Transfusion: exchange if severe
How well did you know this?
1
Not at all
2
3
4
5
Perfectly
26
Q

Old man with smear cells. Cause?

Options

CLL

Hodgkin’s

NHL

A

CLL

Chronic lymphocytic leukaemia is caused by a monoclonal proliferation of well-differentiated lymphocytes which are almost always B-cells (99%)

Features

  • often none
  • constitutional: anorexia, weight loss
  • bleeding, infections
  • lymphadenopathy more marked than CML

Complications

  • hypogammaglobulinaemia leading to recurrent infections
  • warm autoimmune haemolytic anaemia in 10-15% of patients
  • transformation to high-grade lymphoma (Richter’s transformation)

Investigations

  • blood film: smudge cells (also known as smear cells)
  • immunophenotyping
How well did you know this?
1
Not at all
2
3
4
5
Perfectly
27
Q

Guy on ibuprofen started recently for some pain. Comes in with two episodes of vomiting bright red blood. No further episodes. Stable obs, low Hb. What do you do?

Options

Observe for 24 hours and discharge with omeprazole

OGD within 2 hours

OGD on next endoscopy list

A

OGD on next endoscopy list (<24hrs)

Ideally all patients admitted with upper gastrointestinal haemorrhage should undergo Upper GI endoscopy within 24 hours of admission. In those who are unstable this should occur immediately after resuscitation or in tandem with it. The endoscopy department is a potentially dangerous place for unstable patients and it may be safer to perform the endoscopy in theatre with an anaesthetist present.

Management of GI bleed

ABCDE: CIRCULATION!

  • Resuscitate:
    • Head down
    • Airway: Early control of airway is vital (e.g. Drowsy patient with liver failure) -> protect the airway
    • Breathing: 100% O2
    • ​C 2x 14G cannulae:
      • Bloods: cross match blood, check FBC, LFTs (ETOH abuse), U+E (protein meal) and Clotting and ABG (lactate + Hb) and glucose
      • IV crystalloid infusion up to 1L (avoid dilution)
    • D: blood glucose, GCS, Temperature, expose (PR; maleana)
  • Major H’gge protocol: on-going bleeding and haemodynamic instability are likely to require O negative blood pending cross matched blood
    • ​keep Hb >10
  • Upper GI endoscopy (scoring systems)
    • all within 24 hours of admission.
    • If unstable = immediately after resuscitation or in tandem with it.
How well did you know this?
1
Not at all
2
3
4
5
Perfectly
28
Q

Lady collapses on hospital ward. Had co-amox in A&E earlier and reports feeling breathless before collapse, 2 hours later is slightly tachy, slightly hypotensive, resp rate 34, sats 94% on 10L o2. What do you give first?

Options:

Adrenaline

Chlorephenamine

Steroids

IV fluid bolus

A

Adrenaline IM 500 micrograms (0.5ml 1 in 1,000)

hydrocortisone (200mg)

Chlorphenamine (10mg)

Salbutamol (5mg)

How well did you know this?
1
Not at all
2
3
4
5
Perfectly
29
Q

PBC question: antibody test for PBC

A

anti-mitochondrial antibodies (AMA) M2

subtype are present in 98% of patients and are highly specific

Primary biliary cirrhosis (now increasingly referred to as primary biliary cholangitis) is a chronic liver disorder typically seen in middle-aged females (female:male ratio of 9:1). The aetiology is not fully understood although it is thought to be an autoimmune condition. Interlobular bile ducts become damaged by a chronic inflammatory process causing progressive cholestasis which may eventually progress to cirrhosis. The classic presentation is itching in a middle-aged woman
Associations:

  • Sjogren’s syndrome (seen in up to 80% of patients)
  • rheumatoid arthritis
  • systemic sclerosis
  • thyroid disease

Diagnosis

  • anti-mitochondrial antibodies (AMA) M2 subtype are present in 98% of patients and are highly specific
  • smooth muscle antibodies in 30% of patients
  • raised serum IgM

Management

  • pruritus: cholestyramine
  • fat-soluble vitamin supplementation
  • ursodeoxycholic acid
  • liver transplantation e.g. if bilirubin > 100 (PBC is a major indication) - recurrence in graft can occur but is not usually a problem
How well did you know this?
1
Not at all
2
3
4
5
Perfectly
30
Q

PBC questions: histology showing classic PBC

A

Granulomatous

  • PBC: intrahepatic destruction by chronic granulomatous inflammation results in cirrhosis*
  • PSC: inflammation, fibrosis and strictures and intra and extrahepatic ducts*

Primary biliary cirrhosis (now increasingly referred to as primary biliary cholangitis) is a chronic liver disorder typically seen in middle-aged females (female:male ratio of 9:1). The aetiology is not fully understood although it is thought to be an autoimmune condition. Interlobular bile ducts become damaged by a chronic inflammatory process causing progressive cholestasis, which may eventually progress to cirrhosis. The classic presentation is itching in a middle-aged woman

Clinical features

early: may be asymptomatic (e.g. raised ALP on routine LFTs) or fatigue, pruritus

cholestatic jaundice

hyperpigmentation, especially over pressure points

xanthelasmas, xanthomata

also: clubbing, hepatosplenomegaly
late: may progress to liver failure

Complications

malabsorption: osteomalacia, coagulopathy

sicca syndrome occurs in 70% of cases

portal hypertension: ascites, variceal haemorrhage

hepatocellular cancer (20-fold increased risk)

How well did you know this?
1
Not at all
2
3
4
5
Perfectly
31
Q

Guy with metastatic renal cancer has severe pain. is/has vomited 4 times or something. Currently on oral morphine solution prn and helps for about 2 hours. What pain therapy should he be on?

Options:

Fentanyl transdermal

Morphine sub cut infusion

Morphine subcut prn

Oral morph slow release tablets

Oral morph solution prn

A

absence of Renal failure => Morphine sub cut infusion

Fentanyl transdermal Opioids should be used with caution in patients with chronic kidney disease. Alfentanil, buprenorphine and fentanyl are preferred

Vominting, therefore not:

Morphine subcut prn

Oral morph slow release tablets

Oral morph solution prn

How well did you know this?
1
Not at all
2
3
4
5
Perfectly
32
Q

Someone with previous TKR, now tender, febrile, hot, swollen. No evidence of crystals on microscopy. Aspirate showed turbid fluid.

Options:

Septic arthritis

Reactive arthritis

Gout

A

Septic Arthrisis

Symptoms

  • Acutely inflamed tender, swollen joint.
  • ↓ROM
  • Systemically unwell

Investigations

  • Joint aspiration for MCS
  • ↑↑ WCC (e.g. >50,000/mm3) : mostly PMN
  • ↑ESR/CRP, ↑WCC, Blood cultures
  • X-ray

Management

  • IV Abx: vanc + cefotaxime
  • Consider joint washout under GA
  • Splint joint
  • Physiotherapy after infection resolved
How well did you know this?
1
Not at all
2
3
4
5
Perfectly
33
Q

Nutrient deficiency in coeliacs

A

Folate deficiency

Complications of coeliac disease

  • anaemia: iron, folate and vitamin B12 deficiency (folate deficiency is more common than vitamin B12 deficiency in coeliac disease)
  • hyposplenism
  • osteoporosis, osteomalacia (vit D)
  • lactose intolerance
  • enteropathy-associated T-cell lymphoma of small intestine
  • subfertility, unfavourable pregnancy outcomes
  • rare: oesophageal cancer, other malignancies

Coeliac disease is an autoimmune condition caused by sensitivity to the protein gluten. It is thought to affect around 1% of the UK population. Repeated exposure leads to villous atrophy which in turn causes malabsorption. Conditions associated with coeliac disease include dermatitis herpetiformis (a vesicular, pruritic skin eruption) and autoimmune disorders (type 1 diabetes mellitus and autoimmune hepatitis). It is strongly associated with HLA-DQ2 (95% of patients) and HLA-DQ8 (80%).

Signs and symptoms: (indications for coeliac disease screening)

  • Chronic or intermittent diarrhoea
  • Failure to thrive or faltering growth (in children)
  • Persistent or unexplained gastrointestinal symptoms including nausea and vomiting
  • Prolonged fatigue (‘tired all the time’)
  • Recurrent abdominal pain, cramping or distension
  • Sudden or unexpected weight loss
  • Unexplained iron-deficiency anaemia, or other unspecified anaemia

associated condictions: (indications for coeliac disease screening)

  • Autoimmune thyroid disease
  • Dermatitis herpetiformis
  • Irritable bowel syndrome
  • Type 1 diabetes
  • First-degree relatives (parents, siblings or children) with coeliac disease
How well did you know this?
1
Not at all
2
3
4
5
Perfectly
34
Q

Guy who smokes and keeps pigeons, progressive shortness of breath on climbing hills. CXR shows reticulonodular shadowing.

Options

EAA

IPF

Pneumonia

A

Extrinsic allergic alveolitis (EAA, also known as hypersensitivity pneumonitis)

…is a condition caused by hypersensitivity induced lung damage due to a variety of inhaled organic particles. It is thought to be largely caused by immune-complex mediated tissue damage (type III hypersensitivity) although delayed hypersensitivity (type IV) is also thought to play a role in EAA, especially in the chronic phase.

Examples

  • bird fanciers’ lung: avian proteins
  • farmers lung: spores of Saccharopolyspora rectivirgula (formerly Micropolyspora faeni)
  • malt workers’ lung: Aspergillus clavatus
  • mushroom workers’ lung: thermophilic actinomycetes*

Presentation

  • acute: occur 4-8 hrs after exposure, SOB, dry cough, fever
  • chronic

Investigation

  • chest x-ray: upper/mid-zone fibrosis
  • bronchoalveolar lavage: lymphocytosis
  • blood: NO eosinophilia
How well did you know this?
1
Not at all
2
3
4
5
Perfectly
35
Q

Acute presentation of gout. PMH Heart failure, on ramipril. 3rd recurrence. What do you give?

Options:

Naproxen

Colchicine

Allopurinol

A

Colchicine

BNF: Colchicine is an alternative in patients in whom NSAIDs are contra-indicated. Aspirin is not indicated in gout. Allopurinol, febuxostat, and uricosurics are not effective in treating an acute attack and may prolong it indefinitely if started during the acute episode.

The use of colchicine is limited by the development of toxicity at higher doses, but it is of value in patients with heart failure since, unlike NSAIDs, it does not induce fluid retention; moreover, it can be given to patients receiving anticoagulants.

Gout is a form of microcrystal synovitis caused by the deposition of monosodium urate monohydrate in the synovium. It is caused by chronic hyperuricaemia (uric acid > 450 µmol/l)

Acute management

  • NSAIDs
  • intra-articular steroid injection
  • colchicine* has a slower onset of action. The main side-effect is diarrhoea
  • oral steroids may be considered if NSAIDs and colchicine are contraindicated. A dose of prednisolone 15mg/day is usually used
  • if the patient is already taking allopurinol it should be continued

Chronic Management Allopurinol prophylaxis

  • allopurinol should not be started until 2 weeks after an acute attack has settled as it may precipitate a further attack if started too early
  • initial dose of 100 mg od, with the dose titrated every few weeks to aim for a serum uric acid of < 300 µmol/l
  • NSAID or colchicine cover should be used when starting allopurinol
  • Indications
    • recurrent attacks - ‘if a second attack, or further attacks occur within 1 year’
    • tophi
    • renal disease
    • uric acid renal stones
    • prophylaxis if on cytotoxics or diuretics

Lifestyle modifications

  • reduce alcohol intake and avoid during an acute attack
  • lose weight if obese
  • avoid food high in purines e.g. Liver, kidneys, seafood, oily fish (mackerel, sardines) and yeast products

Other points

  • losartan has a specific uricosuric action and may be particularly suitable for the many patients who have coexistant hypertension
  • calcium channel blockers also decrease uric acid levels, possibly by a renal vasodilatory effect
  • increased vitamin C intake (either supplements or through normal diet) may also decrease serum uric acid levels
How well did you know this?
1
Not at all
2
3
4
5
Perfectly
36
Q

Caucasian lady with hands that change from White → blue → red

A

Raynaud’s phenomena

may be primary (Raynaud’s disease) or secondary (Raynaud’s phenomenon)
Raynaud’s disease typically presents in young women (e.g. 30 years old) with symmetrical attacks
Factors suggesting underlying connective tissue disease

  • onset after 40 years
  • unilateral symptoms
  • rashes
  • presence of autoantibodies
  • features which may suggest rheumatoid arthritis or SLE, for example arthritis or recurrent miscarriages
  • digital ulcers, calcinosis
  • very rarely: chilblains

Secondary causes

  • connective tissue disorders: scleroderma (most common), rheumatoid arthritis, SLE
  • leukaemia
  • type I cryoglobulinaemia, cold agglutinins
  • use of vibrating tools
  • drugs: oral contraceptive pill, ergot
  • cervical rib

Management

  • first-line: calcium channel blockers e.g. nifedipine
  • IV prostacyclin infusions: effects may last several weeks/months
How well did you know this?
1
Not at all
2
3
4
5
Perfectly
37
Q

Post transplant patient has dry cough and SOB. Desaturates from 90+ to 83% on exercising. What does he have?

Options:

PCP

TB

A

PCP

…Whilst the organism Pneumocystis carinii is now referred to as Pneumocystis jiroveci, the term Pneumocystis carinii pneumonia (PCP) is still in common use

  • Pneumocystis jiroveci is an unicellular eukaryote, generally classified as a fungus but some authorities consider it a protozoa
  • PCP is the most common opportunistic infection in AIDS
  • all patients with a CD4 count < 200/mm³ should receive PCP prophylaxis

Features

  • dyspnoea
  • dry cough
  • fever
  • very few chest signs

Pneumothorax is a common complication of PCP.
Extrapulmonary manifestations are rare (1-2% of cases), may cause

  • hepatosplenomegaly
  • lymphadenopathy
  • choroid lesions

Investigation

  • CXR: typically shows bilateral interstitial pulmonary infiltrates but can present with other x-ray findings e.g. lobar consolidation. May be normal
  • exercise-induced desaturation
  • sputum often fails to show PCP, bronchoalveolar lavage (BAL) often needed to demonstrate PCP (silver stain shows characteristic cysts)

Management

  • co-trimoxazole
  • IV pentamidine in severe cases
  • steroids if hypoxic (if pO2 < 9.3kPa then steroids reduce risk of respiratory failure by 50% and death by a third)
How well did you know this?
1
Not at all
2
3
4
5
Perfectly
38
Q

Bilateral conductive hearing loss in old lady (Had to interpret Rinne Webers to work it out). Cause?

Options:

Otosclerosis

Meniere’s

Presbyacusis

A

Otosclerosis

Rinne’s test

  • tuning fork is placed over the mastoid process until the sound is no longer heard, followed by repositioning just over external acoustic meatus
  • air conduction (AC) is normally better than bone conduction (BC)
  • if BC > AC then conductive deafness

Otosclerosis describes the replacement of normal bone by vascular spongy bone. It causes a progressive conductive deafness due to fixation of the stapes at the oval window. Otosclerosis is autosomal dominant and typically affects young adults
Onset is usually at 20-40 years - features include:

  • conductive deafness
  • tinnitus
  • normal tympanic membrane*
  • positive family history

Management

  • hearing aid
  • stapedectomy

*10% of patients may have a ‘flamingo tinge’, caused by hyperaemia

Conductive: Impaired conduction anywhere between auricle and
round window.
Canal obstruction: wax, FB
TM perforation: trauma, infection
Ossicle defects: otosclerosis, infection
Fluid in middle ear
SNHL: Defects of cochlea, cohlear N. or brain
Congenital

  • Alports: SNHL + haematuria
  • Jewell-Lange-Nielsen: SNHL + long QT

Acquired

  • Presbyacussis
  • Drugs: gentamicin, vancomycin
  • Infection: meningitis, measles
  • Tumour: vestibular schwannoma

Other: Meniere’s, MS, CPA lesion (e.g. acoustic neuroma), ↓B12

How well did you know this?
1
Not at all
2
3
4
5
Perfectly
39
Q

Someone with URTI symptoms has purple nodules on shins. What does she have?

Erythema nodosum

Erythema multiforme

A

Erythema nodosum

Overview

  • inflammation of subcutaneous fat
  • typically causes tender, erythematous, nodular lesions
  • usually occurs over shins, may also occur elsewhere (e.g. forearms, thighs)
  • usually resolves within 6 weeks
  • lesions heal without scarring

Causes

  • infection: streptococci, TB, brucellosis
  • systemic disease: sarcoidosis, inflammatory bowel disease, Behcet’s
  • malignancy/lymphoma
  • drugs: penicillins, sulphonamides, combined oral contraceptive pill
  • pregnancy
How well did you know this?
1
Not at all
2
3
4
5
Perfectly
40
Q

Commonest cause of corneal ulceration (branching)?

HSV

VZV

HIV

A

HSV

  • Causes: bacterial, herpetic, fungal, protozoa, vasculitic (RA)
    • Dendritic ulcer = Herpes simplex
    • Acanthamoeba: protazoal infection affecting contact lens wearers swimming in pools.

Presentation

  • Pain, photophobia
  • Conjunctival hyperaemia
  • ↓ acuity
  • White corneal opacity

Risk factors: contact lens wearers
Ix: green c¯ fluorescein on slit lamp
Rx: refer immediately to specialist who will

  • Take smears and cultures
  • Abx drops, oral/topical aciclovir
  • Cycloplegics/mydriatics ease photophobia
  • Steroids may worsen symptoms: professionals only

Complications: Scarring and visual loss

Ophthalmic Shingles
Zoster of CNV1
20% of all Shingles (only commoner in thoracic dermatomes)
Presentation

  • Pain in CNV1 dermatome precedes blistering rash
  • 40% → keratitis, iritis
  • Hutchinson’s sign
    • Nose-tip zoster due to involvement of nasociliary branch.
    • ↑ chance of globe involvement as nasociliarry nerve also supplies globe
  • Ophthalmic involvement
    • Keratitis + corneal ulceration (fluorescein stains)
    • ± iritis
How well did you know this?
1
Not at all
2
3
4
5
Perfectly
41
Q

Painful vesicular derm lesion on one side of face around forehead, eye and cheek. Cause?

Options:

VZV

Herpes simplex

A

VZV (or Herpes Zoster)

Shingles is an acute, unilateral, painful blistering rash caused by reactivation of the Varicella Zoster Virus (VZV).

  • Reactivation is due to ↓ immunity/stress
  • Lifetime prevalence = 20%
  • Painful vesicular rash in dermatomal distribution
    • Thoracic and ophthalmic most commonly
      • Herpes zoster ophthalmicus (HZO) describes the reactivation of the varicella zoster virus in the area supplied by the ophthalmic division of the trigeminal nerve. It accounts for around 10% of case of shingles.
    • Multidermatomal / disseminated in immunocomp
    • Ramsay Hunt = ear zoster, facial palsy, ↓ taste, ↓ hearing
  • Rx may ↓ progression to post-herpetic neuralgia <72hrs:
    • Aciclovir PO/IV
    • Famciclovir
    • Valaciclovir
  • Post-Herpetic neuralgia = severe dermatomal pain
    *
How well did you know this?
1
Not at all
2
3
4
5
Perfectly
42
Q

65 yo guy comes to GP with new onset dyspepsia. No weight loss or dysphagia. What do you do first?

  • Options:*
  • H.pylori stool antigen test*

Refer routinely

Refer 2 weeks

OGD.

A

H.pylori stool antigen testing

urgent specialist referral for endoscopic investigation (i.e. seen within 2 weeks) is indicated for patients of any age with dyspepsia when presenting with any of the following:

  • chronic gastrointestinal bleeding
  • progressive unintentional weight loss
  • progressive difficulty swallowing
  • persistent vomiting
  • iron deficiency anaemia
  • epigastric mass or suspicious barium meal
  • nb. also indicated if recent onset rapidly progressive symptoms

Routine endoscopic investigation of patients of any age, presenting with dyspepsia and without alarm signs, is not necessary

  • however, for patients over 55 years old, consider endoscopy when symptoms persist despite Helicobacter pylori (H. pylori) testing and acid suppression therapy, and when patients have one or more of the following: previous gastric ulcer or surgery, continuing need for NSAID treatment or raised risk of gastric cancer or anxiety about cancerReference:
How well did you know this?
1
Not at all
2
3
4
5
Perfectly
43
Q

30+ year old lady with fever and fits. Scan shows temporal lobe necrosis. Cause?

Options:

HSV

Rabies

A

HSV

From the meninges, the virus extends to the adjacent brain where it affects the temporal and inferior frontal lobes first and more severely, and then spreads to the rest of the brain. Adult HSV encephalitis is limited to the brain. Its symptoms are fever, confusion, coma, and seizures. In addition, because of the involvement of the frontal and temporal lobes, patients often display bizarre behaviour, personality changes, anosmia, and gustatory hallucinations. Survivors may have Korsakoff’s amnesia, because of bilateral damage of the hippocampus, dementia, and seizures.

The brain in advanced HSV encephalitis shows diffuse softening and edema, accentuated by hemorrhagic necrosis of the i_nferior frontal and temporal lobes_.

How well did you know this?
1
Not at all
2
3
4
5
Perfectly
44
Q

Ring enhancing lesions on ct brain with HIV?

A

Toxoplasmosis

Toxoplasma gondii is a protozoa which infects the body via the GI tract, lung or broken skin. It’s oocysts release trophozoites which migrate widely around the body including to the eye, brain and muscle. The usual animal reservoir is the cat, although other animals such as rats carry the disease.

Toxoplasmosis

  • accounts for around 50% of cerebral lesions in patients with HIV
  • constitutional symptoms, headache, confusion, drowsiness
  • CT: usually single or multiple ring enhancing lesions, mass effect may be seen
  • management: sulfadiazine and pyrimethamin
How well did you know this?
1
Not at all
2
3
4
5
Perfectly
45
Q

Lady coming to GP with blood test results showing raised prolactin levels and raised IGF-1. Normal TSH, LH, FSH, ACTH. Most likely diagnosis?

Options:

Acromegaly

Macroprolactinoma

TSHoma

A

Acromegaly

In acromegaly there is excess growth hormone secondary to a pituitary adenoma in over 95% of cases. A minority of cases are caused by ectopic GHRH or GH production by tumours e.g. pancreatic

Features

  • coarse facial appearance, spade-like hands, increase in shoe size
  • large tongue, prognathism, interdental spaces
  • excessive sweating and oily skin
  • features of pituitary tumour: hypopituitarism, headaches, bitemporal hemianopia
  • raised prolactin in 1/3 of cases → galactorrhoea
  • 6% of patients have MEN-1

Complications

  • hypertension
  • diabetes (>10%)
  • cardiomyopathy
  • colorectal cancer
  • Growth hormone (GH) levels vary during the day and are therefore not diagnostic. The definitive test is the oral glucose tolerance (OGTT) with serial GH measurements. Serum IGF-1 may also be used as a screening test and is sometimes used to monitor disease

Oral glucose tolerance test

  • in normal patients GH is suppressed to < 2 mu/L with hyperglycaemia
  • in acromegaly there is no suppression of GH
  • may also demonstrate impaired glucose tolerance which is associated with acromegaly
46
Q

Old lady, takes nitrofurantoin - gets watery diarrhea, 2 others in care home have diarrhoea too?

C. difficile

Norovirus

Rotavirus

A

Norovirus

  • SRVS (small round structured virus)
  • Incubation 12–48h mean≈34h
  • Features= Fever, P, D & projectile V; 80 ‘winter vomiting illness’. :
  • no leucocytes in faeces; PCR
  • *Fecal-oral (vomit is infectious); very contagious, and common. Infectious for up to 48h after symptoms resolve

Clostridium Difficile

nitrofurantoin is less likely to cause C.Diff. This is more common with the use of quinolones, cephalosporins, clindamycin and amoxiclav

Presents:

  • Bloody diarrhoea, abdo pain, fever- foul smelling
  • Pseudomembranous colitis
  • Toxic megacolon
  • GI perforation

nb, Secretory diarrhoea:

  • Bacteria only found in lumen: don’t activate innate immunity
  • No / low fever
  • No faecal leukocytes
  • Caused by bacterial toxins: Cholera, E. coli (except EIEC), S. aureus
  • Toxin → ↑cAMP→ open CFTR channel → Cl loss followed by HCO3, Na and H2O loss → secretory diarrhoea
47
Q

Blood transfusion, starts second unit, immediately has chest pain, SOB, dark urine. What happened?

Options:

ABO incompatibility

Immediate haemolytic transfusion reaction

Transfusion associated circulatory overload

Haemolytic transfusion reaction

A

ABO incompatibility

Febrile non-haemolytic transfusion reaction

  • Signs and symptoms: Fever, chills, headache, malaise, flushing
  • Mechanism: host abs vs donor MHC antigens or due to cytokines from leukocytes in donor blood
  • Treatment: discontinue transfusion, fever resolves in 15-30 mins without specific treatment. Paracetamol may be used
  • Other: most common transusion reaction, can be prevented with leukocyte filters or irradiation

Haemolytic transfusion reaction

  • Signs and symptoms: Fever, chills, pain at the infusion site, dark urine, nausea, shock
  • Mechanism: ABO incompatibility with host abs vs antigens on donor RBCs
  • Treatment: discontinue transfusion, administer fluids
  • Other: most SEVERE reaction

Allergic transfusion reaction

  • Signs and symptoms: Uritcaria, pruritus
  • Mechanism: allergic reaction to plasma proteins in transfused blood
  • Treatment: DOES NOT require discontinuing transfusion, symptomatic treatment with antihistamines
  • Other: can be prevented with anti-histamine pre-treatment

Anaphylactic transfusion reaction

  • Signs and symptoms: Uritcaria, angioedema, wheezing, laryngeal oedema, abdo pain, hypotension, shock
  • Mechanism: Host abs vs IgA antibodies in the donor plasma
  • Treatment: discontinue transfusion, administer adrenaline
  • Other: Seen in pts with IgA deficiency, can be prevented by administering washed IgA decifient products

Transfusion relatied acute lung injury (TRALI)

  • Signs and symptoms: dyspnoea, hypoxaemia, bilateral chest infiltrates
  • Mechanism: donor Abs to MHC class I/II/Human neutrophil antigens. Activated neutrophils cause endothelial damage
  • Treatment: discontinue transfusion, provide airway support
  • Other: most common cause of transfusion related DEATH

Transfusion associated circulatory overload

  • Signs and symptoms: dyspnoea, pulmonary oedema, HTN, peripheral oedema
  • Mechanism: rapid volume expansion.
  • Treatment: supportive, diuretics can be used
  • Other: seen in elderly pts with HF/anaemia, can be prevented with slower transfusions and diuretics
48
Q

T1DM girl on subcut insulin pump comes in with frequent episodes of SOB, palpitation and tingling of fingers. Cause?

Options:

Hyperventilation syndrome

Anaemia

Anorexia

A

Hyperventilation syndrome

HVS may present with chest pain and a tingling sensation in the fingertips and around the mouth (paresthesia) and may accompany a panic attack.

49
Q

Most appropriate pre-operative investigation for well-controlled asthmatic about to undergo a diagnostic laparoscopy?

Options:

PEFR

CXR

A

PEFR

50
Q

Guy falls down can’t remember how long he’s been on the floor for, comes in with renal failure and dark red urine, raised potassium. What is cause of renal failure?

A

Rhabdomyolysis

…will typically feature in the exam as a patient who has had a fall or prolonged epileptic seizure and is found to have acute renal failure on admission

Features

  • acute renal failure with disproportionately raised creatinine
  • elevated CK
  • myoglobinuria
  • hypocalcaemia (myoglobin binds calcium)
  • elevated phosphate (released from myocytes)

Causes

  • seizure
  • collapse/coma (e.g. elderly patients collapses at home, found 8 hours later)
  • ecstasy
  • crush injury
  • McArdle’s syndrome
  • drugs: statins

Management

  • IV fluids to maintain good urine output
  • urinary alkalinization is sometimes used
51
Q

Some thin 18 year old girl comes in with low potassium, macrocytic anaemia and raised urea, physical examination otherwise normal. What is cause?

Options:

AN

Coeliac disease

A

AN

Potassium – Hypokalemia is extremely common in anorexia nervosa purging subtype and severe bulimia nervosa

Urea - Low urea is commonin low weight patients due to general malnutrition. A high urea (even with a normal creatinine) is concerning as it may indicate dehydration or renal failure

Macrocytic anaemia - too little vitamin B12 and/or folate

52
Q

Smoker comes in with progressive breathlessness. GP does spirometry and sends him home with PEFR diary. Which will help GP diagnose COPD?

Options:

Long history of smoking

Reversible airflow limitation with beta 2 agonist

Normal FVC and low FEV1

Low FVC and low FEV1

A

Normal FVC and low FEV1

…consider a diagnosis of COPD in patients over 35 years of age who are smokers or ex-smokers and have symptoms such as exertional breathlessness, chronic cough or regular sputum production.

The following investigations are recommended in patients with suspected COPD:

  • post-bronchodilator spirometry to demonstrate airflow obstruction: FEV1/FVC ratio less than 70%
    • FEV1 - significantly reduced
    • FVC - reduced or normal
    • FEV1% (FEV1/FVC) - reduced
  • chest x-ray: hyperinflation, bullae, flat hemidiaphragm. Also important to exclude lung cancer
  • full blood count: exclude secondary polycythaemia
  • body mass index (BMI) calculation
53
Q

Patient with pernicious anaemia then has ?iron def as well and gastroscopy shows ulcer in stomach. What malignancy?

Options:

Lymphoma

Adeno

SCC

Adenosquamous carcinoma

A

Adenocarcinoma

Ulcerative pathophysiology results in adenocarcinoma

  • pernicious anaemia=Gastric carcinoma (adenocarcinoma)
  • H. pylori = Gastric adenocarcinoma and MALT lymphoma
  • Coeliac disease= Intestinal lumphoma

Pernicious anaemia

Epidemiology of pernicious anaemia:

  • more common in females (F:M = 1.6:1)
  • typically develops in middle to old age
  • more common if blood group A

Pathophysiology

  • autoimmune disease caused by antibodies to gastric parietal cells or intrinsic factor
  • results in vitamin B12 deficiency
  • associated with thyroid disease, diabetes, Addison’s, rheumatoid and vitiligo
  • predisposes to gastric carcinoma

Features

  • lethargy, weakness
  • dyspnoea
  • paraesthesia
  • also: mild jaundice, diarrhoea, sore tongue
  • possible signs: retinal haemorrhages, mild splenomegaly, retrobulbar neuritis

Treatment of the disorder is with 3 monthly treatment of vitamin B12 injections. Folic acid supplementation may also be required.

54
Q

Old guy comes in to GP with one episode of visible painless haematuria. What do you do?

Options:

Refer to uro
CT urogram

USS renal tract

A

Refer to urology

refer people using a suspected cancer pathway referral (for an appointment within 2 weeks) for bladder cancer if they are:

  • aged 45 and over and have:
    • unexplained visible haematuria without urinary tract infection or
    • visible haematuria that persists or recurs after successful treatment of urinary tract infection, or
  • aged 60 and over and have unexplained non-visible haematuria and either dysuria or a raised white cell count on a blood test
  • consider non-urgent referral for bladder cancer in people aged 60 and over with recurrent or persistent unexplained urinary tract infection

CT urogram

Transitional cell carcinoma

  • Accounts for 90% of lower urinary tract tumours, but only 10% of renal tumours
  • Males affected 3x more than females
  • Occupational exposure to industrial dyes and rubber chemicals may increase risk
  • Up to 80% present with painless haematuria
  • Diagnosis and staging is with CT IVU
  • TRATMENT: Radical nephroureterectomy
55
Q

Guy comes in with loin to groin pain and blood ++ on urine dip. What ix do you do to confirm?

Options:

CT urogram without contrast

IVU

USS renal tract

A

CT urogram without contrast

Imaging

  • BAUS guidelines recommend ultrasound as the initial imaging modality of choice. The sensitivity of ultrasound for stones is around 45% and specificity is around 90%. Complications such as hydronephrosis can also be quickly identified
  • following an ultrasound, BAUS recommend a non-contrast CT (NCCT) to confirm the diagnosis. 99% of stones are identifiable on NCCT. Some GPs now have direct access to NCCT

Stones < 5 mm will usually pass spontaneously. Lithotripsy and nephrolithotomy may be for severe cases.

56
Q

Guy comes in with lots of swelling and protein in urine and hypoalbumin. What test will confirm a specific diagnosis?

Options:

Renal biopsy

MC&S

A

Renal biopsy

All adults presenting with nephrotic syndrome should recieve a biopsy (in children steroids are sufficient, as the majority of these have minimal change nephropathy)

Cuases of nephrotic syndrome (i.e. Specific diagnosises)

secondary to systemic disease:

  • DM
  • SLE: membranous
  • Amyloidosis

Primary

  1. Minimal Change Glomerulonephritis:
  • commonest cause of nephrotic syndrome in children
  • assoc: URTI
  • Bx: normal light micro, fusion of podocytes on EM
  • Rx; steroids
  • Prog: 1%-> ESRF
  1. Membranous Nephropathy
  • 20-30% of adult nephrotic syndrome
  • Assoc:
    • ca: lung, colon, breast
    • AI: SLE, thyroid disease
    • Infections: HBV
    • Drugs: Penicillamine, gold
  • Bx: subepithelial immune complexes
  • Rx: immunosuppresion if renal dunction declines
  • Progn: 40% spontaneous remision
  1. FSGS:
  • commoner in afrocarribean
  • idiopathic/secondary VUR, Berger’s, SCD, HIV
  • Bx: focal scarring, IgM deposition
  • Rx: steroids or cyclophosphamide/ciclosporin
  • Prog: 30-50% -> ESRF (may occur in transplants!)
  1. Membranoproliferative / Mesangiocapillary GN
  • Rare
  • May-> nephrotic (60%) or nephritic (30%)
  • assoc HBV, HCV, endocarditis
  • Prog: 50%-> ESRF
57
Q

Guy comes in with swollen legs and bilateral scrotal swelling with recent 10kg weight gain?

Options:

Lymphoedema

Peripheral oedema

Hydrocele

A

Peripheral oedema

Lymphoedema implies Ca so more likely see weight loss!

CHF= scrotal oedema and weight gain

58
Q

Lady with metastatic breast ca and hypercalcaemia (like over 3). What do you do first?

Options:

IV fluids

Furosemide

Bisphos

Haemodialysis

A

IV fluids

The initial management of hypercalcaemia is rehydration with normal saline, typically 3-4 litres/day. Following rehydration bisphosphonates may be used. They typically take 2-3 days to work with maximal effect being seen at 7 days

Other options include:

  • calcitonin - quicker effect than bisphosphonates
  • steroids in sarcoidosis

Loop diuretics such as furosemide are sometimes used in hypercalcaemia, particularly in patients who cannot tolerate aggressive fluid rehydration. However, they should be used with caution as they may worsen electrolyte derangement and volume depletion.

59
Q

Guy with hypercalcaemia, renal failure, anaemia, bone pain. What does he have?

Options:

MM

Secondary hyper pth

A

Multiple myeloma

….is a neoplasm of the bone marrow plasma cells. The peak incidence is patients aged 60-70 years.

Clinical features

  • bone disease: bone pain, osteoporosis + pathological fractures (typically vertebral), osteolytic lesions
  • lethargy (anaemia)
  • infection
  • hypercalcaemia (see below)
  • renal failure
  • other features: amyloidosis e.g. Macroglossia, carpal tunnel syndrome; neuropathy; hyperviscosity

Investigations

  • monoclonal proteins (usually IgG or IgA) in the serum and urine (Bence Jones proteins)
  • increased plasma cells in the bone marrow
  • historically a skeletal survey has been done to look for bone lesions. However, whole-body MRI is increasingly used and is now recommended in the 2016 NICE guidelines

The diagnostic criteria for multiple myeloma requires one major and one minor criteria or three minor criteria in an individual who has signs or symptoms of multiple myeloma.

Major criteria:

  • Plasmacytoma (as demonstrated on evaluation of biopsy specimen)
  • 30% plasma cells in a bone marrow sample
  • Elevated levels of M protein in the blood or urine

Minor criteria

  • 10% to 30% plasma cells in a bone marrow sample.
  • Minor elevations in the level of M protein in the blood or urine.
  • Osteolytic lesions (as demonstrated on imaging studies).
  • Low levels of antibodies (not produced by the cancer cells) in the blood.

Hypercalcaemia in myeloma

primary factor: due primarily to increased osteoclastic bone resorption caused by local cytokines (e.g. IL-1, tumour necrosis factor) released by the myeloma cells

much less common contributing factors: impaired renal function, increased renal tubular calcium reabsorption and elevated PTH-rP levels

60
Q

Lady who recently came from Afghanistan with low calcium, raised PTH. What test to confirm Dx?

Serum vitamin D

A

Serum vitamin D

Osteomalacia

  • normal bony tissue but decreased mineral content
  • rickets if when growing
  • osteomalacia if after epiphysis fusion

Types

  • vitamin D deficiency e.g. malabsorption, lack of sunlight, diet
  • renal failure
  • drug induced e.g. anticonvulsants
  • vitamin D resistant; inherited
  • liver disease, e.g. cirrhosis

Features

  • rickets: knock-knee, bow leg, features of hypocalcaemia
  • osteomalacia: bone pain, fractures, muscle tenderness, proximal myopathy

Investigation

  • low calcium, phosphate, 25(OH) vitamin D
  • raised alkaline phosphatase
  • raised pTH
  • x-ray: children - cupped, ragged metaphyseal surfaces; adults - translucent bands (Looser’s zones or pseudofractures)

Treatment: calcium with vitamin D tablets

61
Q

Guy with epilepsy and now housebound. Pain in spine and on compressing rib cage? What does he have?

Options:

Osteomalacia

Osteoarthritis

Polychondritis

A

Osteomalacia

62
Q

Guy has episodes of syncope and and goes blue during. No other problems. How to investigate?

Options:

24 hour ECG

Echo

Angiography

A

24 hour ECG

63
Q

Someone with right sided hemiparesis, aphasia, some kind of visual deficit - doesn’t blink to hand waving on right side but does on left. Which artery?

Options:

MCA

ACA

Basilar

Posterior circulation

A

MCA

Classic example of right parietal stoke, most often involved the parietal lobule. The artery infarcted is the R MCA. Because the MCA continues in a nearly straight line from the ICA, is it a common route for small emboli originating from the ica.

The parietal lobs is responsible for the primary and sesnosry information. One of the types of sensory information provided by this region is the ability to localise objects in space. People with lesions in the right parietal lobe live in ‘right-sided worlds’ and ignore the left dise of their bodies and all objects in space. Because the fibres from the lateral fields of the optic tract run through the parietal lobe of the contralateral side, a visual defect often accompanies strokes in this region and patients don’t blink in response to rand-waving in this field.

64
Q

Bitemporal hemianopia person where is lesion. She’s almost knocking pedestrians down.

Optic chiasm

A

Optic chiasm

Bitemporal hemianopia:

  • lesion of optic chiasm
  • upper quadrant defect > lower quadrant defect = inferior chiasmal compression, commonly a pituitary tumour
  • lower quadrant defect > upper quadrant defect = superior chiasmal compression, commonly a craniopharyngioma

Other visual field defects:

The main points for the exam are:

  • left homonymous hemianopia means visual field defect to the left, i.e. Lesion of right optic tract
  • homonymous quadrantanopias: PITS (Parietal-Inferior, Temporal-Superior)
  • incongruous defects = optic tract lesion; congruous defects = optic radiation lesion or occipital cortex

A congruous defect simply means complete or symmetrical visual field loss and conversely an incongruous defect is incomplete or asymmetric. Please see the link for an excellent diagram.

Homonymous hemianopia

  • incongruous defects: lesion of optic tract (differently shaped defect in both eyes)
  • congruous defects: lesion of optic radiation or occipital cortex (similarly shaped defect in both eyes)
  • macula sparing: lesion of occipital cortex

Homonymous quadrantanopias*

  • superior: lesion of temporal lobe
  • inferior: lesion of parietal lobe
  • mnemonic = PITS (Parietal-Inferior, Temporal-Superior)
65
Q

Fit guy whose dad died early comes to you worried. He passes out during sports i think. What investigation?

Options:

Echo

ECG

Refer to cardiology

A

ECG

Initially: ECG (shows structural and ?arythmic changes)

  • ECG:
    • left ventricular hypertrophy, often gross
    • ischaemic changes e.g. T wave inversion
  • chest X-ray:
    • normal until left ventricular dilatation in heart failure
  • echocardiography is generally diagnostic, showing:
    • septal hypertrophy
    • mid-systolic closure of aortic valve
    • an anterior movement of mitral valve during systole
  • 24 hour tape - patients at risk of arrhythmias

HOCM: nb also arrhythmogenic right ventricular dysplasia (and mixed)

Echo - mnemonic - MR SAM ASH

  • mitral regurgitation (MR)
  • systolic anterior motion (SAM) of the anterior mitral valve leaflet
  • asymmetric hypertrophy (ASH)

ECG

  • left ventricular hypertrophy
  • progressive T wave inversion
  • deep Q waves
  • atrial fibrillation may occasionally be seen

Arrhythmogenic right ventricular dysplasia (and mixed)

Right ventricular myocardium is replaced by fatty and fibrofatty tissue
Around 50% of patients have a mutation of one of the several genes which encode components of desmosome
ECG abnormalities in V1-3, typically T wave inversion. An epsilon wave is found in about 50% of those with ARV - this is best described as a terminal notch in the QRS complex

66
Q

Guy falls down from somewhere and has no more biceps reflex, hand held in (basically Erb’s palsy position but this is described out in excruciating detail without using the words Erb’s palsy). What got damaged?

Options:

Upper brachial plexus

Lower brachial plexus

Median

Ulnar

Radial

A

Upper Brachial Plexus

Erb-Duchenne palsy (‘waiter’s tip’)

  • due to damage of the upper trunk of the brachial plexus (C5,C6)
  • may be secondary to shoulder dystocia during birth
  • the arm hangs by the side and is internally rotated, elbow extended
67
Q

Zig zag lines and painful eyes+ N&V for 30 min episodes ?

Options:

No cluster headache option.

Migraine

Glaucoma

A

Migraine

  • Recurrent, severe headache which is usually unilateral and throbbing in nature
  • May be be associated with aura, nausea and photosensitivity
    • zig zag lines
  • Aggravated by, or causes avoidance of, routine activities of daily living. Patients often describe ‘going to bed’.
  • In women may be associated with menstruation

Acute angle closure glaucoma:

  • severe pain: may be ocular or headache
  • decreased visual acuity
  • symptoms worse with mydriasis (e.g. watching TV in a dark room)
  • hard, red eye
  • haloes around lights
  • semi-dilated non-reacting pupil
  • corneal oedema results in dull or hazy cornea
  • systemic upset may be seen, such as nausea and vomiting and even abdominal pain
68
Q

Guy with haemorrhoids that do not prolapse. First line?

Options:

Fibre

Sclerotherapy

Surgery

Topical GTN

A

Fibre

Haemorrhoidal tissue is part of the normal anatomy which contributes to anal continence. These mucosal vascular cushions are found in the left lateral, right posterior and right anterior portions of the anal canal (3 o’clock, 7’o’clock and 11 o’clock respectively). Haemorrhoids are said to exist when they become enlarged, congested and symptomatic

Clinical features

  • painless rectal bleeding is the most common symptom
  • pruritus
  • pain: usually not significant unless piles are thrombosed
  • soiling may occur with third or forth degree piles

Types of haemorrhoids

  • External
    • originate below the dentate line
    • prone to thrombosis, may be painful
  • Internal
    • originate above the dentate line
    • do not generally cause pain

Grading of internal haemorrhoids

  • Grade I: Do not prolapse out of the anal canal
  • Grade II: Prolapse on defecation but reduce spontaneously
  • Grade III: Can be manually reduced
  • Grade IV: Cannot be reduced

Management

  • soften stools: increase dietary fibre and fluid intake
  • topical local anaesthetics and steroids may be used to help symptoms
  • outpatient treatments: rubber band ligation is superior to injection sclerotherapy
  • surgery is reserved for large symptomatic haemorrhoids which do not respond to outpatient treatments
  • newer treatments: Doppler guided haemorrhoidal artery ligation, stapled haemorrhoidopexy

Acutely thrombosed external haemorrhoids

  • typically present with significant pain
  • examination reveals a purplish, oedematous, tender subcutaneous perianal mass
  • if patient presents within 72 hours then referral should be considered for excision. Otherwise patients can usually be managed with stool softeners, ice packs and analgesia. Symptoms usually settle within 10 days
69
Q

High esr, pain on abducting shoulder?

Options

PMR

RA

A

Polymyalgia rheumatica

Pathophysiology

  • overlaps with temporal arteritis
  • histology shows vasculitis with giant cells, characteristically ‘skips’ certain sections of affected artery whilst damaging others
  • muscle bed arteries affected most in polymyalgia rheumatica

Features

  • typically patient > 60 years old
  • usually rapid onset (e.g. < 1 month)
  • aching, morning stiffness in proximal limb muscles (not weakness)
  • also mild polyarthralgia, lethargy, depression, low-grade fever, anorexia, night sweats

Investigations

  • ESR > 40 mm/hr
  • note CK and EMG normal
  • reduced CD8+ T cells

Treatment: prednisolone e.g. 15mg/od - dramatic response

70
Q

Patent ductus arteriosus murmur. What is it?

Options:

Machinery murmur

Diastolic murmur

Venous hum

A

Machinery murmur

Patent Ductus Arteriosis

  • a form of congenital heart defect
  • generally classed as ‘acyanotic’. However, uncorrected can eventually result in late cyanosis in the lower extremities, termed differential cynaosis.
  • connection between the pulmonary trunk and descending aorta
  • more common in premature babies, born at high altitude or maternal rubella infection in the first trimester

Features

  • left subclavicular thrill
  • continuous ‘machinery’ murmur
  • large volume, bounding, collapsing pulse
  • wide pulse pressure
  • heaving apex beat

Management

  • indomethacin closes the connection in the majority of cases
  • if associated with another congenital heart defect amenable to surgery then prostaglandin E1 is useful to keep the duct open until after surgical repair
71
Q

Old person with painful swollen knee and now has swollen dip.?

Options:

Osteoarthritis

Rheumatoid arthritis

A

Osteoarthritis

Bouchards nodes: Hard, bony outgrowths or gelatinous cysts on the proximal interphalangeal joints (the middle joints of fingers or toes.) They are a sign of osteoarthritis, and are caused by formation of calcific spurs of the articular cartilage. (NB: RA is a disease of the synovium, DIPs are not synovial!)

Heberdens nodes: Typically develop in middle age, beginning either with a chronic swelling of the affected joints or the sudden painful onset of redness, numbness, and loss of manual dexterity. This initial inflammation and pain eventually subsides, and the patient is left with a permanent bony outgrowth that often skews the fingertip sideways.

72
Q

Patient has dislocated shoulder. What nerve should we watch out for?

options:

Axillary

Radial

Ulnar

A

Axillary nerve

  • Axillary nerve* (C5,C6): Motor=Shoulder abductions (deltoid) I Sensory=Inferior region of the Deltoid muscle I Injured=humeral neck #/dislocation, Result=deltoid wasting
  • Radial nerve* (C5-C8): Motor=Extension (forearm, wrist, fingers, thumb) I Sensory= Small area between the dorsal aspect of the 1st and 2nd metacarpals I Injured=Humeral midshaft fracture, Result=Palsy results in wrist drop
  • Median nerve* (C6, C8, T1): Motor=LOAF* muscles, @wrist=paralysis of thenar muscle, opponens pollicis, @elbow=loss of pronation of forearem and weak wrist flexion I Sensory=Palmar aspect of lat 3.1/2 fingers I Injured=wrist lesion (CTS)
  • Ulnar nerve* (C8, T1): Motor=Intrinsic hand muscles except LOAF* and wrist flexion I Sensory=Medial 1.1/2 fingers. I Injured= medial epicondyle #, damage may result in claw hand
  • Musculocutaneous nerve* (C5-C7): Motor=Elbow flexion (supplies biceps brachii) and supination I Sensory=Lateral part of the forearm I Injured=Isolated injury rare - usually injured as part of brachial plexus injury
  • Long thoracic nerve* (C5-C7): Motor=Serratus anterior I Injured=Often during sport e.g. following a blow to the ribs. Also possible complication of mastectomy. Result= in a winged scapula
73
Q

Lady with fever with catheter in situ 14 days after some bowel surgery. Eating and drinking normally, no resp or GI symptoms. Cause of fever?

Options:

UTI

Anastomotic leak

Basal atelectasis

Pneumonia

Wound infection

A

Urinary Tract Infection

Risk due to instumentation

Early causes of post-op pyrexia (0-5 days) include:

  • Blood transfusion
  • Cellulitis
  • Urinary tract infection
  • Physiological systemic inflammatory reaction (usually within a day following the operation)
  • Pulmonary atelectasis - this if often listed but the evidence base to support this link is limited

Late causes (>5 days) include:

  • Venous thromboembolism
  • Pneumonia: resp signs
  • Wound infection
  • Anastomotic leak: Generalised sepsis causing mediastinitis or peritonitis depending on site of leak
74
Q

Patient has long term indwelling suprapubic catheter. Keeps getting blocked and flushed out by nurses a few times. What do you do?

Options:

Flush

Bladder washout

Change suprapubic catheter

Change to other catheter

TURP

A

Bladder washout

75
Q

ENT question on purulent otorrhoea, hearing loss, some reference to keratin. Cause?

Options:

Cholesteatoma

Recurrent otitis externa

Rcurrent otitis media

A

Cholesteatoma

consists of squamous epithelium that is ‘trapped’ within the skull base causing local destruction. It is most common in patients aged 10-20 years.

Main features

  • foul smelling discharge
  • hearing loss

Other features are determined by local invasion:

  • vertigo
  • facial nerve palsy
  • cerebellopontine angle syndrome

Otoscopy: ‘attic crust’ - seen in the uppermost part of the ear drum
Management: patients are referred to ENT for consideration of surgical remova

76
Q

Lady comes back from india 3 days ago with abdo pain, diarrhoea and vomiting. What do you do?

Options:

Stool culture and microscopy

Colonoscopy

Start abx

A

Stool culture and microscopy

Gastroenteritis may either occur whilst at home or whilst travelling abroad (travellers’ diarrhoea)

Travellers’ diarrhoea may be defined as at least 3 loose to watery stools in 24 hours with or without one of more of abdominal cramps, fever, nausea, vomiting or blood in the stool. The most common cause is Escherichia coli

Another pattern of illness is ‘acute food poisoning’. This describes the sudden onset of nausea, vomiting and diarrhoea after the ingestion of a toxin. Acute food poisoning is typically caused by Staphylococcus aureus, Bacillus cereus or Clostridium perfringens.

Incubation period

  • 1-6 hrs: Staphylococcus aureus, Bacillus cereus*
  • 12-48 hrs: Salmonella, Escherichia coli
  • 48-72 hrs: Shigella, Campylobacter
  • > 7 days: Giardiasis, Amoebiasis
77
Q

Hypertensive Young guy with mildly raised cr and proteinuria on multiple occasions. Physical exam otherwise unremarkable. What does he have?

Options:

Orthostatic proteinuria

Amyloid

Hypertensive Nephropathy

A

Nephritic Syndrome??Hypertensive Nephropathy

Orthostatic proteinuria is a benign condition

78
Q

Lady about to undergo ovarian tumour resection and needs thromboprophylaxis but had a PE in the past. What do you give?

Options:

LMWH

Unfractionated heparin

High dose warfarin

Low dose warfarin

A

LMWH

Deep vein thrombosis may develop insidiously in many surgical patients. Untreated it may progress to result in pulmonary embolism.

The following surgical patients are at increased risk of deep vein thrombosis:

  • Surgery greater than 90 minutes at any site or greater than 60 minutes if the procedure involves the lower limbs or pelvis
  • Acute admissions with inflammatory process involving the abdominal cavity
  • Expected significant reduction in mobility
  • Age over 60 years
  • Known malignancy
  • Thrombophilia
  • Previous thrombosis
  • BMI >30
  • Taking hormone replacement therapy or the contraceptive pill
  • Varicose veins with phlebitis

Mechanical thromboprophylaxis

  • Early ambulation after surgery is cheap and is effective
  • Compression stockings (contra -indicated in peripheral arterial disease)
  • Intermittent pneumatic compression devices
  • Foot impulse devices

Low molecular weight heparin: MODA=Binds antithrombin resulting in inhibition of factor Xa. Notes=In patients with normal renal function, low doses typically given in those with moderate to high risk of thromboembolic events. It is given as once daily subcutaneous injection

Unfractionated heparin: MODA=Binds antithrombin III with affects thrombin and factor Xa. Notes=Effective anticoagulation, administered intravenously it has a rapid onset and its therapeutic effects decline quickly on stopping and infusion. Its activity is measured using the APTT. If need be it can be reversed using protamine sulphate

Dabigatran: MODA=Orally administered direct thrombin inhibitor. Notes=Used prophylaxis in hip and knee surgery. It does not require therapeutic monitoring. It has no known antidote and should not be used in any patient in whom there is a risk of active bleeding or imminent likelihood of surgery

79
Q

Something about a guy with a droopy face including forehead??

Options:

Bell’s

UMN lesion

A

Bell’s Palsy

….may be defined as an acute, unilateral, idiopathic, facial nerve paralysis. The aetiology is unknown although the role of the herpes simplex virus has been investigated previously. The peak incidence is 20-40 years and the condition is more common in pregnant women.

Features

  • lower motor neuron facial nerve palsy - forehead affected*
  • patients may also notice post-auricular pain (may precede paralysis), altered taste, dry eyes, hyperacusis

Management

  • in the past a variety of treatment options have been proposed including no treatment, prednisolone only and a combination of aciclovir and prednisolone
  • following a National Institute for Health randomised controlled trial it is now recommended that prednisolone 1mg/kg for 10 days should be prescribed for patients within 72 hours of onset of Bell’s palsy. Adding in aciclovir gives no additional benefit
  • eye care is important - prescription of artificial tears and eye lubricants should be considered

Prognosis: if untreated around 15% of patients have permanent moderate to severe weakness

80
Q

What is most appropriate osteoporosis prophylaxis for 65 year old lady on long term pred?

Options:

Bisphos

Vit D

HRT

Calcium

A

Bisphosphonate

Management of patients at risk of corticosteroid-induced osteoporosis

  1. Patients over the age of 65 years or those who’ve previously had a fragility fracture should be offered bone protection.
  2. Patients under the age of 65 years should be offered a bone density scan, with further management dependent:

The first-line treatment is alendronate. Patients should also be calcium and vitamin D replete.

81
Q

Post mastect + LN clearance lady gets winging of scapula. What nerve damaged?

Options:

Long thoracic

Axillary

A

Long thoracic

Long thoracic nerve (C5-C7): Motor=Serratus anterior I Injured=Often during sport e.g. following a blow to the ribs. Also possible complication of mastectomy. Result= in a winged scapula

Axillary nerve (C5,C6): Motor=Shoulder abductions (deltoid) I Sensory=Inferior region of the Deltoid muscle I Injured=humeral neck #/dislocation, Result=deltoid wasting

Radial nerve (C5-C8): Motor=Extension (forearm, wrist, fingers, thumb) I Sensory= Small area between the dorsal aspect of the 1st and 2nd metacarpals I Injured=Humeral midshaft fracture, Result=Palsy results in wrist drop

Median nerve (C6, C8, T1): Motor=LOAF* muscles, @wrist=paralysis of thenar muscle, opponens pollicis, @elbow=loss of pronation of forearem and weak wrist flexion I Sensory=Palmar aspect of lat 3.1/2 fingers I Injured=wrist lesion (CTS)

Ulnar nerve (C8, T1): Motor=Intrinsic hand muscles except LOAF* and wrist flexion I Sensory=Medial 1.1/2 fingers. I Injured= medial epicondyle #, damage may result in claw hand

Musculocutaneous nerve (C5-C7): Motor=Elbow flexion (supplies biceps brachii) and supination I Sensory=Lateral part of the forearm I Injured=Isolated injury rare - usually injured as part of brachial plexus injury

82
Q

Patient with lung ca findings + cushingoid features. Heavy smoker. What’s wrong?

Options:

Small cell

Adeno

Squamous cell carcinoma

A

Small cell

Paraneoplastic syndrome

Small cell

  • ADH: SIADH
  • ACTH - not typical, hypertension, hyperglycaemia, hypokalaemia, alkalosis and muscle weakness are more common than buffalo hump etc
  • Lambert-Eaton syndrome

Squamous cell

  • parathyroid hormone-related protein (PTH-rp) secretion causing hypercalcaemia
  • clubbing
  • hypertrophic pulmonary osteoarthropathy (HPOA)
  • hyperthyroidism due to ectopic TSH

Adenocarcinoma

  • gynaecomastia
  • hypertrophic pulmonary osteoarthropathy (HPOA)
83
Q

Someone with Hodgkins lymphoma, mediastinal mass and night sweats. What is most appropriate treatment?

Options:

Curative chemo

Adjuvant chemo

Neoadjuvant chemo

Radio

Surgery

A

Curative chemotherapy

Staging (Ann Arbor system) Influences treatment and prognosis. Done by CXR, CT/PET of thorax, abdo, pelvis ± marrow biopsy if B symptoms, or stage III–IV disease.

  1. I Confined to single lymph node region.
  2. II Involvement of two or more nodal areas on the same side of the diaphragm.
  3. III Involvement of nodes on both sides of the diaphragm.
  4. IV Spread beyond the lymph nodes, eg liver or bone marrow.

Each stage is either ‘A’—no systemic symptoms other than pruritus; or ‘B’—presence of B symptoms: weight loss >10% in last 6 months, unexplained fever >38°C, or night sweats (needing change of clothes).

  • favourable early stage disease
    • with the current available data, the standard of care for treatment of patients with early stage favourable HL includes combination modality therapy using adriamycin, bleomycin, vinblastine, and decarbazine (ABVD) for 2 cycles and 20 Gy radiotherapy
  • unfavourable early stage disease
    • currently four cycles of ABVD followed by 30 Gy RT is widely considered the standard of care for unfavourable early stage HL
    • another treatment option for unfavourable early stage HL is 2 cycles of bleomycin/etoposide/adriamycin/cyclophosphamide/vincristine/procarb (BEACOPP) escalated doses + 2 x ABVD and 30Gy RT
  • advanced stage disease
    • patients aged 16 to 60 with advanced stage HL should receive either 6-8 cycles of ABVD or 6 cycles of escalated BEACOPP
    • the European Society for Medical Oncology (ESMO) recommend localised radiation with 30 Gy to residual lymphoma greater than 1.5cm
84
Q

Old lady gets dysphagia to solids every now and again, 20 year hx of GORD.?

Options:

Stricture

Ca

Achalasia

A

Stricture

Peptic stricture: Longer history of dysphagia, often not progressive. Usually symptoms of GORD. Often lack systemic features seen with malignancy, weight loss, anorexia or vomiting during eating

Adenocarcinoma: Progressive dysphagia, may have previous symptoms of GORD or Barretts oesophagus.

Achalasia: Dysphagia of both liquids and solids from the start. Heartburn. Regurgitation of food - may lead to cough, aspiration pneumonia etc

Squamous cell carcinoma: History of progressive dysphagia. Often signs of weight loss. Usually little or no history of previous GORD type symptoms. But Hx of smoking/drinking?

85
Q

Some standard change in bowel habit + weight loss + fresh blood mixed in with stool. Dilation of colon proximal to sigmoid. Diagnosis?

Options:

CRC

Sigmoid volvulus

A

CRC

A bit about screening…

The following patients should be referred urgently (i.e. within 2 weeks) to colorectal services for investigation:

  • patients >= 40 years with unexplained weight loss AND abdominal pain
  • patients >= 50 years with unexplained rectal bleeding
  • patients >= 60 years with iron deficiency anaemia OR change in bowel habit

An urgent referral (within 2 weeks) should be ‘considered’ if:

  • there is a rectal or abdominal mass
  • there is an unexplained anal mass or anal ulceration
  • patients < 50 years with rectal bleeding AND any of the following unexplained symptoms/findings:
    • -→ abdominal pain
    • -→ change in bowel habit
    • -→ weight loss
    • -→ iron deficiency anaemia

National screening programme offering screening every 2 years to all men and women aged 60 to 74 years. Patients aged over 74 years may request screening.
In addition FOBT should be offered to:

  • patients >= 50 years with unexplained abdominal pain OR weight loss
  • patients < 60 years with changes in their bowel habit Or iron deficiency anaemia
  • patients >= 60 years who have anaemia even in the absence of i_ron deficiency_

*

86
Q

Someone has bloody stool. Proctosigmoidoscopy is normal. What do you do next?

Options:

Colonoscopy

Red cell uptake scan

A

Colonoscopy

87
Q

Patient with hx of biliary colic, now presenting with jaundice and obstructive picture. What is diagnosis?

Options:

Gallstone in CBD

Pancreatic ca

A

Gallstone in CBD

88
Q

Lady with collapse High K, Low Na, plus some other bloods suggesting addisons. What would get you the Dx?

A

ACTH and cortisol levels (/Short synACTHen)

In a patient with suspected Addison’s disease the definite investigation is a ACTH stimulation test (short Synacthen test). Plasma cortisol is measured before and 30 minutes after giving Synacthen 250ug IM. Adrenal autoantibodies such as anti-21-hydroxylase may also be demonstrated.

If a ACTH stimulation test is not readily available (e.g. in primary care) then sending a 9 am serum cortisol can be useful:

  • > 500 nmol/l makes Addison’s very unlikely
  • < 100 nmol/l is definitely abnormal
  • 100-500 nmol/l should prompt a ACTH stimulation test to be performed

Associated electrolyte abnormalities are seen in around one-third of undiagnosed patients:

  • hyperkalaemia
  • hyponatraemia
  • hypoglycaemia
  • metabolic acidosis
89
Q

What to put in an alcoholics drip when he comes in confused and ataxic and shit. Repeated from yesterday.?

Options:

Thiamine

Chlordiazepoxide.

A

Thiamine

Confusion + Ataxia (+nystagmus)= wernicke’s encephalopathy = thiamin vit B1 defficiency

Thiamine is therefore important in the catabolism of sugars and aminoacids. The clinical consequences of thiamine deficiency are therefore seen first in highly aerobic tissues such as the brain (Wenicke-Korsakoff syndrome) and the heart (wet beriberi).
Causes of thiamine deficiency:

  • alcohol excess
  • malnutrition

Alcohol withdrawal

  • Mechanism
    • chronic alcohol consumption enhances GABA mediated inhibition in the CNS (similar to benzodiazepines) and inhibits NMDA-type glutamate receptors
    • alcohol withdrawal is thought to be lead to the opposite (decreased inhibitory GABA and increased NMDA glutamate transmission)
  • Features
    • symptoms start at 6-12 hours: tremor, sweating, tachycardia, anxiety
    • peak incidence of seizures at 36 hours
    • peak incidence of delirium tremens is at 48-72 hours: coarse tremor, confusion, delusions, auditory and visual hallucinations, fever, tachycardia
  • Management
    • first-line: benzodiazepines e.g. chlordiazepoxide. Typically given as part of a reducing dose protocol
    • carbamazepine also effective in treatment of alcohol withdrawal
    • phenytoin is said not to be as effective in the treatment of alcohol withdrawal seizures
90
Q

Patient gets pain to right of epigastrium a few hours after food. Relieved by milk and wakes up at night with pain?

Options:

Duodenal ulcer

Gastric ulcer

Gastritis

A

Duodenal ulcer

PUD: Epigastric pain

  • There may be a history of NSAID use or alcohol excess.
  • Duodenal ulcers: more common than gastric ulcers, epigastric pain relieved by eating
  • Gastric ulcers: epigastric pain worsened by eating
  • Features of upper gastrointestinal haemorrhage may be seen (haematemesis, melena etc)
91
Q

Guy with history of delirium tremens. Wants to quit again. What do you do for him?

Options:

Reducing regime of chlordiazepoxide

Refer to AA

Refer to local specialist alcohol services

A

Refer to local specialist alcohol services

^In a patient with a history of DT it is important that they are carefully monitored

92
Q

Someone with Sjogren’s, which test do you do?

Anti-Ro

Sialography

Biopsy

A

anti-Ro? or Biopsy

[GP Notebook, Sjogren’s syndrome is diagnosed if there is more than one focus where there are more than 50 lymphocytes per 4 mm2 in the salivary tissues]

Sjogren’s syndrome is an autoimmune disorder affecting exocrine glands resulting in dry mucosal surfaces. It may be primary (PSS) or secondary to rheumatoid arthritis or other connective tissue disorders, where it usually develops around 10 years after the initial onset. Sjogren’s syndrome is much more common in females (ratio 9:1). There is a marked increased risk of lymphoid malignancy (40-60 fold)

Features

  • dry eyes: keratoconjunctivitis sicca
  • dry mouth
  • vaginal dryness
  • arthralgia
  • Raynaud’s, myalgia
  • sensory polyneuropathy
  • renal tubular acidosis (usually subclinical)

Investigation

  • rheumatoid factor (RF) positive in nearly 100% of patients
  • ANA positive in 70%
  • anti-Ro (SSA) antibodies in 70% of patients with PSS
  • anti-La (SSB) antibodies in 30% of patients with PSS
  • Schirmer’s test: filter paper near conjunctival sac to measure tear formation
  • histology: focal lymphocytic infiltration
  • also: hypergammaglobulinaemia, low C4

Management

  • artificial saliva and tears
  • pilocarpine may stimulate saliva production
93
Q

Someone with SLE described, which test do you do?

A

Anti-dsDNA (=specific and sensitive), anti- Smith (=most specific, less sensitive)

Immunology

  • 99% are ANA positive
  • 20% are rheumatoid factor positive
  • anti-dsDNA: highly specific (> 99%), but less sensitive (70%)
  • anti-Smith: most specific (> 99%), sensitivity (30%)
  • also: anti-U1 RNP, SS-A (anti-Ro) and SS-B (anti-La)

Monitoring

  • ESR: during active disease the CRP is characteristically normal - a raised CRP may indicate underlying infection
  • complement levels (C3, C4) are low during active disease (formation of complexes leads to consumption of complement)
  • anti-dsDNA titres can be used for disease monitoring (but note not present in all patients)
94
Q

Post op lady with sx consistent with DVT. What test confirms diagnosis?

Options:

Venous duplex

Ultrasound

Coagulation screen.

A

Venous duplex

DVT Investigation

Wells score

Clinical probability simplified score
If a DVT is ‘likely’ (2 points or more)

  • a proximal leg vein ultrasound scan should be carried out within 4 hours and, if the result is negative, a D-dimer test
  • if a proximal leg vein ultrasound scan cannot be carried out within 4 hours a D-dimer test should be performed and low-molecular weight heparin administered whilst waiting for the proximal leg vein ultrasound scan (which should be performed within 24 hours)

If a DVT is ‘unlikely’ (1 point or less)

  • perform a D-dimer test and if it is positive arrange:
  • a proximal leg vein ultrasound scan within 4 hours
  • if a proximal leg vein ultrasound scan cannot be carried out within 4 hours low-molecular weight heparin should be administered whilst waiting for the proximal leg vein ultrasound scan (which should be performed within 24 hours)
95
Q

Which cardiac drug contraindicated in asthma?

A

atenolol

CI in asthma:

  • B-blockers e.g. atenolol, carvidilol
  • NSAIDs (with the excesption of aspirin)
    • NB: If asthmatic patient had no wheeze with ibuprofen, may continue it – don’t necessarily have to stop it, as means asthma not NSAID sensitive
96
Q

One about someone with CXR pleural plaques, presenting with picture of pleural effusion, breathlessness. What ix helps with diagnosis?

Options:

USS guided tap

Bronchoscopy

A

USS guided tap

(?could be asbestos/mesothelioma)

and send aspirate to Chemistry, microbiology and cytology

Pleural effusions may be classified as being either a transudate or exudate according to the protein concentration.
Transudate (< 30g/L protein)

  • heart failure (most common transudate cause)
  • hypoalbuminaemia (liver disease, nephrotic syndrome, malabsorption)
  • hypothyroidism
  • Meigs’ syndrome

Exudate (> 30g/L protein)

  • infection: pneumonia (most common exudate cause), TB, subphrenic abscess
  • connective tissue disease: RA, SLE
  • neoplasia: lung cancer, mesothelioma, metastases
  • pancreatitis
  • pulmonary embolism
  • Dressler’s syndrome
  • yellow nail syndrome

Features

  • dyspnoea, non-productive cough or chest pain are possible presenting symptoms
  • classic examination findings include dullness to percussion, reduced breath sounds and reduced chest expansion
97
Q

Guy with pulseless electrical activity, given one dose of adrenaline/epinephrine. Still no carotid pulse. What now?

Options:

Another dose of adrenaline

Shock

Pericardiocentiesis

A

Another dose of adrenaline

Guidance for a non-shockable rhythm: Give adrenaline every 3-5minutes

Pericardiocentiesis: only if indicated (US)

98
Q

One with a patient with RTA then complete white out of one side of lung.

Options

Haemothorax

Fail chest

Tamponade

A

Haemothorax

Differential for ‘Life threatening chest injury’: ATOMIC

  • Airway obstruction
  • Tension Pneumothorax
  • Open pneumothorax (sucking)
  • Massive haemothorax
  • Intercostal disruption and pulmonary contusion
  • Cardiac Tamponade

Massive Haemothorax

  • Accumulation of >1.5L of blood in chest cavity
  • Usually caused by disruption of hilar vessels
  • Presentation:
    • Signs of chest wall trauma
    • ↓BP
    • ↓ expansion
    • ↓ breath sounds and ↓VR
    • Stony dull percussion
  • Mx
    • X-match 6u
    • Large-bore chest drain c¯ hep saline for autotransfusion
    • Thoracotomy if >1.5L or >200ml/h
99
Q

There was another vasc one about someone who had something that sounded like leaking AAA. What is ix you would do immediately?

Options:

FBC

CT abdo

Arteriogram

A

CT abdo

Because of the varying symptoms and signs of aortic dissection depending on the initial intimal tear and the extent of the dissection, the proper diagnosis is sometimes difficult to make.

While taking a good history from the individual may be strongly suggestive of an aortic dissection, the diagnosis cannot always be made by history and physical signs alone. Often, the diagnosis is made by visualization of the intimal flap on a diagnostic imaging test. Common tests used to diagnose an aortic dissection include a CT scan of the abdomen.

Investigations:

  • Bloods: FBC, cross-match 10 units of blood, U&E (renal function), clotting.
  • CT-abdomen: False lumen of dissection can be visualized.
100
Q

Guy with hx of BPH, IHD, indigestion, diabetes on metformin, simvastatin, bisoprolol, bunch of other drugs, has erectile dysfunction. All pulses present. Normal peripheral sensation. LSBP no orthostatic hypotension. Cause?

Options:

Adverse drug effect

BPH

Autonomic neuropathy

Vascular insufficiency

A

Adverse drug effect

Drugs which may occasionally cause may erectile dysfunction include:

  • phenothiazines
  • benzodiazepines
  • butyrophenones
  • tricyclic antidepressants
  • monoamine oxidase inhibitors

Antihypertensive drugs

  • diuretics
  • beta-blockers

Others

  • finasteride
  • cimetidine
  • oestrogens
  • antiandrogens
101
Q

Young guy with poor stream, takes ages to pee, previous episode of non-specific urethritis. Cause?

Options:

Detrussor dysfunction

Urethral stricture

A

Urethral stricture

102
Q

Opioid overdose treatment

A

Naloxone

103
Q

Chest pain with nausea vomiting and sweating for 1 hour. What was it?

A

MI

104
Q

DKA

Options:

giving IV fluids with IV insulin

SC continuous insulin

A

giving IV fluids with IV insulin

Management

  • fluid replacement: most patients with DKA are deplete around 5-8 litres. Isotonic saline is used initially. Please see an example fluid regime below.
  • insulin: an intravenous infusion should be started at 0.1 unit/kg/hour. Once blood glucose is < 15 mmol/l an infusion of 5% dextrose should be started
  • correction of hypokalaemia
105
Q

Marathon runner has a large heart found on post mortem?

Options:

Hypertrophy

Hyperplasia

Metaplasia

Dysplasia

A

Exercise induced hypertrophic cardiomyopathy

106
Q

RA patient with previous TKR has red hot swollen knee, recently had cystoscopy for longstanding urinary tract problems.

Options:

Infection

Pseudogout

A

Infection

107
Q

38 year old lady with cyclical breast pain.

Options:

Abnormal normal development and involution of breast (ANDI)

Mastitis

Fibrocystic disease

A

ANDI

Aberrations in the Normal Development and Involution of the breast (ANDI) is an all-encompassing term that is used to describe a wide spectrum of the benign breast diseases. As the name suggests, it is based on the theory that most of the encountered benign breast disorders are essentially minor aberrations in the normal development process, hormonal response and involution of the breast

Fibroadenoma
Under the age of 25 years the breast is usually classified as undergoing development. Lobular units are being formed and a dense stroma is formed within the breast tissue. This may result in the development of fibroadenomas.
As a group fibroadenomas account for 13% of all palpable breast lesions. However, in women aged 18-25 they constitute up to 60% of all palpable breast lesions. The are classified as juvenile, common and giant. The former occur in early adolescence and the latter are characterised by a size greater than 4cm. In young females with small fibroadenomas (less than 3cm on imaging) a policy of watchful waiting without biopsy may be adopted. A size of greater than 4cm attracts a recommendation for core biopsy to exclude a phyllodes tumour. The natural history of fibroadenomas is that 10% will increase in size, 30% regress and the remainder stay the same. This does not apply during pregnancy and lactation when they may increase in size substantially and subsequently sequester milk.
Some women may wish to have their fibroadenomas excised, they can usually be shelled out through a circumareolar incision. Smaller lesions may be removed using a mammotome.

Breast cysts
Palpable cysts constitute 15% of all breast lumps. They occur most frequently in perimenopausal females and are caused by distended and involuted lobules.
They may be readily apparent on clinical examination as soft, fluctuant swellings. It is important to exclude the presence of an underlying mass lesion. On imaging they will usually show a ‘halo appearance’ on mammography. Ultrasound will confirm the fluid filled nature of the cyst. Symptomatic cysts may be aspirated and following aspiration the breast re-examined to ensure that the lump has gone.

Duct ectasia
As women progress through the menopause the breast ducts shorten and dilate. In some women this may cause a cheese like nipple discharge and slit like retraction of the nipple. No specific treatment is required.

108
Q

Patient with asthma has a short hx of yellow sputum and wheeze, was given oral amox which improved sputum but wheeze still there but afebrile, what now?

Options:

Give prolonged course of amox

Give clari

Give oral pred

A

Oral prednisolone

Rx Wheeze

109
Q

Patient has carotid endarterectomy and develops stridor post-op.

Options:

Haemorrhage

False aneurysm

A

Haemorrhage

  • > Haematoma
  • > stridor

Also: Risk of damage to the Hypoglossal nerve and sympathetic plexus

110
Q

Man newly diagnosed with DM found to have glucose+++ in urine, BMI 32, random blood glucose 14. First line?

Options:

Diet and exercise

Metformin

Gliclazide

A

Dietary advice

  • encourage high fibre, low glycaemic index sources of carbohydrates
  • include low-fat dairy products and oily fish
  • control the intake of foods containing saturated fats and trans fatty acids
  • limited substitution of sucrose-containing foods for other carbohydrates is allowable, but care should be taken to avoid excess energy intake
  • discourage use of foods marketed specifically at people with diabetes
  • initial target weight loss in an overweight person is 5-10%

Next=Metformin

111
Q

Why is MR spine the best imaging choice in someone with disc prolapse

Options:

Better soft tissue detail

Better cortical bone detail

A

Better soft tissue detail

112
Q

Lady presents with acute MI, no details. What do you give?

Options:

Aspirin and alteplase

Aspirin and beta blockers

A

Aspirin and alteplase

In the absence of contraindications, all patients should be given

  • aspirin
  • P2Y12-receptor antagonist. Clopidogrel / ticagrelor / prasugrel
  • unfractionated heparin (pt going for PCI) Alternatives = LMWH
  • THEN:
    • PCI (but is not available in all centres)
    • Thrombolysis
      • alteplase/tenecteplase